Medical Laboratory Science Review- Harr - Immunology

¡Supera tus tareas y exámenes ahora con Quizwiz!

A pregnant woman came to her physician with a maculopapular rash on her face and neck. Her temperature was 37.7°C (100°F). Rubella tests for both IgG and IgM antibody were positive. What positive test(s) would reveal a diagnosis of congenital rubella syndrome in her baby after birth? A. Positive rubella tests for both IgG and IgM antibody B. Positive rubella test for IgM C. Positive rubella test for IgG D. No positive test is revealed in congenital rubella syndrome

B A finding of IgG is not definitive for congenital rubella syndrome because IgG crosses the placenta from the mother; however, demonstration of IgM, even in a single neonatal sample, is diagnostic.

Which immunoglobulin appears in highest titer in the secondary response? A. IgG B. IgM C. IgA D. IgE

A A high titer of IgG characterizes the secondary immune response. Consequently, IgG antibodies comprise about 80% of the total immunoglobulin concentration in normal serum.

A streptozyme test was performed, but the result was negative, even though the patient showed clinical signs of a streptococcal throat infection. What should be done next? A. Either ASO or anti-deoxyribonuclease B (anti-DNase B) testing B. Another streptozyme test using diluted serum C. Antihyaluronidase testing D. Wait for 3-5 days and repeat the streptozyme test

A A streptozyme test is used for screening and contains several of the antigens associated with streptococcal products. Because some patients produce an antibody response to a limited number of streptococcal products, no single test is sufficiently sensitive to rule out infection. Clinical sensitivity is increased by performing additional tests when initial results are negative. The streptozyme test generally shows more false positives and false negatives than ASO and anti-DNase. A positive test for antihyaluronidase occurs in a smaller number of patients with recent streptococcal infections than ASO and anti-DNase.

Interpret the following ASO results: Tube Nos. 1-4 (Todd unit 125): no hemolysis; Tube No. 5 (Todd unit 166): hemolysis A. Positive Todd unit 125 B. Positive Todd unit 166 C. No antistreptolysin O present D. Impossible to interpret

A An ASO titer is expressed in Todd units as the last tube that neutralizes (no visible hemolysis) the streptolysin O (SLO). Most laboratories consider an ASO titer significant if it is 166 Todd units or higher. However, people with a recent history of streptococcal infection may demonstrate an ASO titer of 166 or higher; demonstration of a rise in titer from acute to convalescent serum is required to confirm a current streptococcal infection. ASO is commonly measured using a rapid latex agglutination assay. These tests show agglutination when the ASO concentration is 200 IU/mL or higher.

How is complement activity destroyed in vitro? A. Heating serum at 56oC for 30 min B. Keeping serum at room temperature of 22oC for 1 hour C. Heating serum at 37oC for 45 min D. Freezing serum at 0oC for 24 hours

A complement activity in serum in vitro is destroyed by heating the serum at 56oC fir 30 min. In test procedures where complement may interfere with the test system, it may be necessary to destroy complement activity in the test sample by heat inactivation.

A carbohydrate antigen 125 assay (CA-125) was performed on a woman with ovarian cancer. After treatment, the levels fell significantly. An examination performed later revealed the recurrence of the tumor, but the CA 125 levels remained low. How can this finding be explained? A. Test error B. CA-125 was the wrong laboratory test; α-fetoprotein (AFP) is a better test to monitor ovarian cancer C. CA-125 may not be sensitive enough when used alone to monitor tumor development D. CA-125 is not specific enough to detect only one type of tumor

2. C CA-125 is a tumor associated carbohydrate antigen that is elevated in 70%-80% of patients with ovarian cancer and about 20% of patients with pancreatic cancer. While an increase in CA-125 may indicate recurrent or progressive disease, failure to do so does not necessarily indicate the absence of tumor growth.

What is the correct procedure upon receipt of a test request for human chorionic gonadotropin (hCG) on the serum from a 60-year-old man? A. Return the request; hCG is not performed on men B. Perform a qualitative hCG test to see if hCG is present C. Perform the test; hCG may be increased in testicular tumors D. Perform the test but use different standards and controls

3. C hCG is normally tested for in pregnancy; it is increased in approximately 60% of patients with testicular tumors and a lower percentage of those with ovarian, GI, breast, and pulmonary tumors. Malignant cells secreting hCG may produce only the β-subunit; therefore, qualitative and quantitative tests that detect only intact hormone may not be appropriate.

An 18-month-old boy has recurrent sinopulmonary infections and septicemia. Bruton's X-linked immunodeficiency syndrome is suspected. Which test result would be markedly decreased? A. Serum IgG, IgA, and IgM B. Total T-cell count C. Both B- and T-cell counts D. Lymphocyte proliferation with phytohemagglutinin stimulation

A A patient with Bruton's X-linked agammaglobulinemia presents with clinical symptoms related to recurrent infections, demonstrated in the laboratory by decreased or absent immunoglobulins. Peripheral blood B cells are absent or markedly reduced, but T cells are normal in number and function. Because phytohemagglutinin is a T-cell mitogen, the lymphocyte proliferation test using PHA would be normal for this patient.

Which test should be performed when a patient has a reaction to transfused plasma products? A. Immunoglobulin levels B. T-cell count C. Hemoglobin levels D. Red cell enzymes

A A reaction to plasma products may be found in an IgA-deficient person who has formed anti-IgA antibodies. Immunoglobulin levels would aid in this determination. Selective IgA deficiency is the most common immunodeficiency disease and is characterized by serum IgA levels below 5 mg/dL. IgA is usually absent from secretions, but the B-cell count is usually normal.

Select the best donor for a man, blood type AB, in need of a kidney transplant. A. His brother, type AB, HLA matched for class II antigens B. His mother, type B, HLA matched for class I antigens C. His cousin, type O, HLA matched for major class II antigens D. Cadaver donor, type O, HLA matched for some class I and II antigens

A A twin or sibling donor of the same blood type and HLA matched for class II antigens is the best donor in this situation. Class II antigens (HLA-D, HLA-DR, DQ, and DP) determine the ability of the transplant recipient to recognize the graft. The HLA genes are located close together on chromosome 6, and crossover between HLA genes is rare. Siblings with closely matched class II antigens most likely inherited the same class I genes. The probability of siblings inheriting the same HLA haplotypes from both parents is 1:4.

Which disease is least likely when a nucleolar pattern occurs in an immunofluorescence test for antinuclear antibodies? A. MCTD B. Sjögren's syndrome C. SLE D. Scleroderma

A All of the diseases except MCTD may cause a nucleolar pattern of immunofluorescence. Nucleolar fluorescence is caused by anti-RNA antibodies and is seen in about 50% of patients with scleroderma.

A patient with ovarian cancer who has been treated with chemotherapy is being monitored for recurrence using serum CA-125, CA-50, and CA 15-3. Six months after treatment the CA 15-3 is elevated, but the CA-125 and CA-50 remain low. What is the most likely explanation of these findings? A. Ovarian malignancy has recurred B. CA 15-3 is specific for breast cancer and indicates metastatic breast cancer C. Testing error occurred in the measurement of CA 15-3 caused by poor analytical specificity D. The CA 15-3 elevation is spurious and probably benign

A Although CA-125 is the most commonly used tumor marker for ovarian cancer, not all ovarian tumors produce CA-125. Greatest sensitivity in monitoring for recurrence is achieved when several markers known to be increased in the malignant tissue type are measured simultaneously and when the markers are elevated (by malignancy) prior to treatment. In addition to limited sensitivity, no single tumor marker is entirely specific. Carbohydrate and other oncofetal antigens are produced by several malignant and benign conditions. Although testing errors may occur in any situation, measurements of carbohydrate antigens use purified monoclonal antibodies with very low cross reactivities.

A patient with joint swelling and pain tested negative for serum RF by both latex agglutination and ELISA methods. What other test would help establish a diagnosis of RA in this patient? A. Anti CCP B. ANA testing C. Flow cytometry D. Complement levels

A Antibodies to cyclic citruillinated peptide are often found in RF-negative patients with rheumatoid arthritis. The absence of rheumatoid factors from serum does not rule out a diagnosis of RA, and more than half of patients who are diagnosed with RA present initially with a negative serum result. The serum RF test will eventually be positive in 80%-90% of patients who meet the clinical criteria for RA.

A specimen appears to have a perinuclear staining pattern in an antineutrophil cytoplasmic antibody (ANCA) immunofluorescent assay using ethanol- fixed neutrophils, suggesting the possibility of a pANCA. On which of the following substrates would this specimen display cytoplasmic speckling? A. Formalin-fixed neutrophils B. Unfixed neutrophils C. HEp-2 cells D. Rabbit kidney tissue

A Antibodies to neutrophil cytoplasmic antigen demonstrating a perinuclear pattern of fluorescence indicate a diagnosis of vasculitis. However, atypical ANCAs and ANAs also demonstrate a perinuclear staining pattern on ethanol-fixed neutrophils. To differentiate these from pANCA, specimens appearing as a pANCA on ethanol-fixed cells are tested on formalin-fixed neutrophils. The myeloperoxidase containing granules that coalesce around the nuclear membrane during ethanol fixation will remain in the cytoplasm during formalin fixation. Thus, pANCA will have a cytoplasmic (cANCA) pattern on a formalin-fixed slide, but ANAs will retain a perinuclear pattern and the fluorescence will be diminished.

Antibodies to thyroid peroxidase can be detected by using agglutination assays. Which of the following diseases may show positive results with this type of assay? A. Graves' disease and Hashimoto's thyroiditis B. Myasthenia gravis C. Granulomatous thyroid disease D. Addison's disease

A Antibodies to thyroid peroxidase may be detected in both Graves' disease (hyperthyroidism) and Hashimoto's thyroiditis (hypothyroidism). If a positive result is found to thyroid peroxidase, thyroxine levels can be measured to distinguish between the two diseases.

An ANA test on HEp-2 cells shows nucleolar staining in interphase cells and dense chromatin staining in mitotic cells. The most likely cause of this staining pattern is: A. Antifibrillarin antibody B. Antiribosomal p antibody C. A serum with nucleolar and homogeneous patterns D. Technical artifact

A Antifibrillarin antibody has this appearance. Ribosomal p antibody has nucleolar staining and a background homogeneous and cytoplasmic stain. A combination nucleolar/homogeneous specimen will also show homogeneous staining in the interphase cells. This pattern is not seen in typical technical artifacts.

What type of antibodies is represented by the solid or homogeneous pattern in the immunofluorescence test for antinuclear antibodies? A. Antihistone antibodies B. Anticentromere antibodies C. Anti-ENA (anti-Sm and anti-RNP) antibodies D. Anti-RNA antibodies

A Antihistone antibodies (and also anti-DNA antibodies) cause the solid or homogeneous pattern, which is commonly found in patients with SLE, RA, mixed connective tissue disease, and Sjögren's syndrome. Antibodies to the centromere of chromosomes is a marker for the CREST (calcinosis, Raynaud's phenomenon, esophageal dysfunction, sclerodactyly, and telangiectasia) form of systemic sclerosis.

A patient was tested for syphilis by the RPR method and was reactive. An FTA-ABS test was performed and the result was negative. Subsequent testing showed the patient to have a high titer of anticardiolipin antibodies (ACAs) by the ELISA method. Which routine laboratory test is most likely to be abnormal for this patient? A. Activated partial thromboplastin time (APTT) B. Antismooth muscle antibodies C. Aspartate aminotransferase (AST) D. C3 assay by immunonephelometry

A Approximately 50%-70% of patients with ACA also have the lupus anticoagulant (LAC) in their serum. he LAC is an immunoglobulin that interferes with in vitro coagulation tests: prothrombin time (PT), APTT, and dilute Russell's viper venom (DRVV) time. These tests require phospholipid for the activation of factor X. About 30% of patients with antibodies to cardiolipin or phospholipids have a biological false-positive RPR result. Antismooth muscle is most commonly associated with chronic active hepatitis, and increased AST with necrotic liver diseases. Although ACA and LAC may be associated with SLE, the majority of patients with these antibodies do not have lupus and would have a normal C3 level.

Which of the following methods used for HIV identification is considered a signal amplification technique? A. Branched chain DNA analysis B. DNA PCR C. Reverse transcriptase PCR D. Nucleic acid sequence based assay (NASBA)

A Branched chain DNA is a signal amplification technique, i.e., if you start with one copy of the gene you finish with one copy. The detection reagent is amplified, increasing the sensitivity of the assay.

A patient deficient in the C3 complement component would be expected to mount a normal: A. Type I and IV hypersensitivity response B. Type II and IV hypersensitivity response C. Type I and III hypersensitivity response D. Type II and III hypersensitivity response

A Complement is involved in types II and III hypersensitivity; thus an individual deficient in C3 will be deficient in those responses. The complement deficiency should have no effect on IgE (type I) or cell-mediated (type IV) hypersensitivities.

A patient who is blood group O is accidentally transfused with group A blood and develops a reaction during the transfusion. What antibody is involved in this type II reaction? A. IgM B. IgE C. IgG and IgE D. IgG

A IgG and IgM are the antibodies involved in a type II cytotoxic reaction. Naturally occurring anti-A in the form of IgM is present in the blood of a group O individual and would cause an immediate transfusion reaction. Cell destruction occurs when antibodies bind to cells causing destruction via complement activation, thereby triggering intravascular hemolysis.

Which of the following conditions will most likely result in a false-negative DAT test? A. Insufficient washing of RBCs B. Use of heavy chain-specific polyclonal anti-human Ig C. Use of excessive centrifugal force D. Use of a sample obtained by finger puncture

A Insufficient washing can cause incomplete removal of excess or unbound immunoglobulins and other proteins, which may neutralize the antiglobulin reagent.

Which of the following substances, sometimes used as a tumor marker, is increased two- or threefold in a normal pregnancy? A. Alkaline phosphatase (ALP) B. Calcitonin C. Adrenocortocotropic hormone (ACTH) D. Neuron-specific enolase

A Isoenzymes of ALP are sometimes used as tumor markers but have a low specificity because they are also increased in nonmalignant diseases. These include the placental-like (heat-stable) ALP isoenzymes, which are found (infrequently) in some malignancies such as cancer of the lung; bone-derived ALP, which is a marker for metastatic bone cancer; and the fast-migrating liver isoenzyme, which is a marker for metastatic liver cancer. ACTH is secreted as an ectopic hormone in some patients with cancer of the lung. Calcitonin is a hormone produced by the medulla of the thyroid and is increased in the serum of patients with medullary thyroid carcinoma. Neuron-specific enolase is an enzyme that is used as a tumor marker primarily for neuroblastoma.

Why is laboratory diagnosis difficult in cases of Lyme disease? A. Clinical response may not be apparent upon initial infection; IgM antibody may not be detected until 3-6 weeks after the infection B. Laboratory tests may be designed to detect whole Borrelia burgdorferi, not flagellar antigen found early in infection C. Most laboratory tests are technically demanding and lack specificity D. Antibodies formed initially to B. burgdorferi may cross react in antigen tests for autoimmune diseases

A Lyme disease is caused by B. burgdorferi, a spirochete, and typical clinical symptoms such as rash or erythema chronicum migrans may be lacking in some infected individuals. Additionally, IgM antibody is not detectable by laboratory tests until 3-6 weeks after a tick bite, and IgG antibody develops later.

A newborn is to be tested for a vertically transmitted HIV infection. Which of the following tests is most useful? A. HIV PCR B. CD4 count C. Rapid HIV antibody test D. HIV IgM antibody test

A Neonatal HIV diagnosis is performed by screening for the presence of the virus. The current antibody tests are either IgG-specific or an IgG/IgM combination assay. Thus an infant whose mother is HIV positive will also be positive in the HIV antibody assay. Although the CD4 count may be a useful assay to determine disease activity, there are many causes of reduced CD4 numbers and this assay should not be used to diagnose HIV infection.

What is an advantage of performing a prostate- specific antigen (PSA) test for prostate cancer? A. PSA is stable in serum and not affected by a digital-rectal examination B. PSA is increased only in prostatic malignancy C. A normal serum level rules out malignant prostatic disease D. The percentage of free PSA is elevated in persons with malignant disease

A PSA is a glycoprotein with protease activity that is specific for the prostate gland. High levels may be caused by prostate malignancy, benign prostatic hypertrophy, or prostatitis, but PSA is not increased by physical examination of the prostate. PSA has a sensitivity of 80% and a specificity of about 75% for prostate cancer. The sensitivity is sufficiently high to warrant its use as a screening test, but sensitivity for stage A cancer is below 60%. Most of the serum PSA is bound to protease inhibitors such as α1-antitrypsin and α1-antichymotrypsin. Patients with borderline PSA levels (4-10 ng/mL) and a low percentage of free PSA are more likely to have cancer of the prostate than patients with a normal percentage of free PSA.

Which test would best distinguish between SLE and MCTD? A. Multiplex or ELISA test for anti-SM and anti-RNP B. Immunofluorescence testing using Crithidia as substrate C. Slide agglutination testing D. Laboratory tests cannot distinguish between these disorders

A The Ouchterlony (double) immunodiffusion assay may be used to identify and differentiate anti-Sm from anti-RNP. Multiplex and ELISA assays, using purified or recombinant antigens, are also available for this testing. Anti-Sm with or without anti-RNP is found in approximately one third of SLE patients. Anti-RNP in the absence of anti-Sm is found in over 95% of MCTD patients.

A patient came to his physician complaining of a rash, severe headaches, stiff neck, and sleep problems. Laboratory tests of significance were an elevated sedimentation rate (ESR) and slightly increased liver enzymes. Further questioning of the patient revealed that he had returned from a hunting trip in upstate New York 4 weeks ago. His physician ordered a serological test for Lyme disease, and the assay was negative. What is the most likely explanation of these results? A. The antibody response is not sufficient to be detected at this stage B. The clinical symptoms and laboratory results are not characteristic of Lyme disease C. The patient likely has an early infection with hepatitis B virus D. Laboratory error has caused a false-negative result

A The antibody response to B. burgdorferi may not develop until several weeks after initial infection. The antibody test should be followed by a test such as PCR to detect the DNA of the organism. Regardless of the test outcome, if the physician suspects Lyme disease, treatment should begin immediately.

A patient receives a transfusion of packed red cells and fresh frozen plasma and develops an anaphylactic, nonhemolytic reaction. She reports receiving a transfusion 20 years earlier. She had no reaction to the previous transfusion, but she did feel "poorly" a few weeks later. Which of the following transfused substances most likely elicited the reaction? A. IgA B. Group A antigen C. Rho (D) antigen D. An antigen belonging to the Duffy system

A The fact that this is a nonhemolytic reaction suggests that a non-red cell antigen may be involved. Selective IgA deficiency occurs in approximately 1 in 700 individuals and is often asymptomatic. Individuals deficient in IgA may make an antibody against the α heavy chain if they are exposed to IgA via a transfusion. This antibody may lead to a serum sickness reaction if the IgA is still present after antibody formation. This could explain the "poor feeling" the patient had after the initial transfusion. A subsequent transfusion may lead to an Arthus reaction if IgG anti-IgA is present or an anaphylactic reaction if IgE anti-IgA is present.

What screening test should be performed first in a young patient suspected of having an immune dysfunction disorder? A. Complete blood count (CBC) and white cell differential B. Chemotaxis assay C. Complement levels D. Bone marrow biopsy

A The first screening tests performed in the initial evaluation of a young patient who is suspected of having an immune dysfunction are the CBC and differential. White blood cells that are decreased in number or abnormal in appearance may indicate further testing.

What type of disorders would show a decrease in C3, C4, and CH50? A. Autoimmune disorders such as SLE and RA B. Immunodeficiency disorders such as common variable immunodeficiency C. Tumors D. Bacterial, viral, fungal, or parasitic infections

A The pattern of decreased C3, C4, and CH50 indicates classic pathway activation. This results in consumption of complement and is associated with SLE, serum sickness, subacute bacterial endocarditis, and other immune complex diseases. The inflammatory response seen in malignancy and acute infections gives rise to an increase in complement components. Immunodeficiency caused by an inherited deficiency in complement constitutes only about 1% of immunodeficiency diseases. Such disorders reduce the CH50 but involve a deficient serum level of only one complement factor.

Which of the following is a description of a type I hypersensitivity reaction? A. Ragweed antigen cross links with IgE on the surface of mast cells, causing release of preformed mediators and resulting in symptoms of an allergic reaction B. Anti-Fya from a pregnant woman crosses the placenta and attaches to the Fya antigen-positive red cells of the fetus, destroying the red cells C. Immune complex deposition occurs on the glomerular basement membrane of the kidney, leading to renal failure D. Exposure to poison ivy causes sensitized T cells to release lymphokines that cause a localized inflammatory reaction

A Type I immediate hypersensitivity (anaphylactic) responses are characterized by IgE molecules binding to mast cells via the Fc receptor. Cross linking of surface IgE caused by binding of allergens causes the mast cell to degranulate, releasing histamine and other chemical mediators of allergy. Answer B describes a type II reaction; C describes a type III reaction; and D describes a type IV reaction.

A 19-year-old girl came to her physician complaining of a sore throat and fatigue. Upon physical examination, lymphadenopathy was noted. Reactive lymphocytes were noted on the differential, but a rapid test for IM antibodies was negative. Liver enzymes were only slightly elevated. What test(s) should be ordered next? A. Hepatitis testing B. EBV serological panel C. HIV confirmatory testing D. Bone marrow biopsy

B An EBV serological panel would give a more accurate assessment than a rapid slide IM test. The time of appearance of the various antibodies to the viral antigens differs according to the clinical course of the infection.

Which disease might be indicated by antibodies to smooth muscle? A. Atrophic gastritis B. Autoimmune hepatitis C. Myasthenia gravis D. Sjögren's syndrome

B Antibodies to smooth muscle are found in the serum of up to 70% of patients with active chronic hepatitis and up to 50% of patients with primary biliary cirrhosis.

What is a general definition for autoimmunity? A. Increase of tolerance to self-antigens B. Loss of tolerance to self-antigens C. Increase in clonal deletion of mutant cells D. Manifestation of immunosuppression

B Autoimmunity is a loss of tolerance to self-antigens and the subsequent formation of autoantibodies.

A patient had surgery for colorectal cancer, after which he received chemotherapy for 6 months. The test for carcinoembryonic antigen (CEA) was normal at this time. One year later, the bimonthly CEA was elevated (above 10 ng/mL). An examination and biopsy revealed the recurrence of a small tumor. What was the value of the results provided by the CEA test in this clinical situation? A. Diagnostic information B. Information for further treatment C. Information on the immunologic response of the patient D. No useful clinical information in this case

B CEA is a glycoprotein that is elevated in about 60% of patients with colorectal cancer and one third or more patients with pulmonary, gastric, and pancreatic cancers. CEA may be positive in smokers, patients with cirrhosis, Crohn's disease, and other nonmalignant conditions. Because sensitivity for malignant disease is low, CEA is not recommended for use as a diagnostic test. However, an elevated CEA after treatment is evidence of tumor recurrence and the need for second-look surgery.

How can interfering cold agglutinins be removed from a test sample? A. Centrifuge the serum and remove the top layer B. Incubate the clot at 1°C-4°C for several hours, then remove serum C. Incubate the serum at 56°C in a water bath for 30 minutes D. Use an anticoagulated sample

B Cold agglutinins will attach to autologous red cells if incubated at 1°C-4°C. The absorbed serum will be free of cold agglutinins.

Which method, classically used for HLA-D typing, is often used to determine the compatibility between a living organ donor and recipient? A. Flow cytometry B. Mixed lymphocyte culture (MLC) C. Primed lymphocyte test (PLT) D. Restriction fragment length polymorphism (RFLP)

B Flow cytometry can be used in transplantation to type serologically defined HLA antigens. The one-way mixed lymphocyte reaction is used to identify HLA-D antigens on the donor's lymphocytes and is used for cross matching living donors with transplant recipients. The assay is time consuming and would not be used as part of a workup for a cadaver donor transplant. HLA-D incompatibility is associated with the recognition phase of allograft rejection. The primed lymphocyte test is used to identify HLA-DP antigens.

How is HLA typing used in the investigation of genetic diseases? A. For prediction of the severity of the disease B. For genetic linkage studies C. For direct diagnosis of disease D. Is not useful in this situation

B HLA typing is useful in predicting some genetic diseases and for genetic counseling because certain HLA types show strong linkage to some diseases. HLA typing is not specifically used to diagnose a disease or assess its severity. In linkage studies, a disease gene can be predicted because it is located next to the locus of a normal gene with which it segregates. For example, the relative risk of developing ankylosing spondylitis is 87% in persons who are positive for HLA-B27. Analysis of family pedigrees for the linkage marker and disease can be used to determine the probability that a family member will inherit the disease gene.

Which of the following positive antibody tests may be an indication of recent vaccination or early primary infection for rubella in a patient with no clinical symptoms? A. Only IgG antibodies positive B. Only IgM antibodies positive C. Both IgG and IgM antibodies positive D. Fourfold rise in titer for IgG antibodies

B If only IgM antibodies are positive, this result indicates a recent vaccination or an early primary infection.

A child suspected of having an inherited humoral immunodeficiency disease is given diphtheria/ tetanus vaccine. Two weeks after the immunization, his level of antibody to the specific antigens is measured. Which result is expected for this patient if he/she indeed has a humoral deficiency? A. Increased levels of specific antibody B. No change in the level of specific antibody C. An increase in IgG-specific antibody but not IgM-specific antibody D. Increased levels of nonspecific antibody

B In an immunodeficient patient, the expected levels of specific antibody to the antigens in the vaccine would be decreased or not present. This response provides evidence of deficient antibody production.

SITUATION: Cells type negative for all HLA antigens in a complement-dependent cytotoxicity assay. What is the most likely cause? A. Too much supravital dye was added B. Rabbit complement is inactivated C. All leukocytes are dead D. Antisera is too concentrated

B Inactive rabbit complement may not become fixed to antibodies that have bound test leukocytes; therefore, no lysis of cells will occur. When the supravital dye is added, all cells will appear negative (exclude the dye) for all HLAs.

Would an hCG test using a monoclonal antibody against the β-subunit of hCG likely be affected by an increased level of follicle-stimulating hormone (FSH)? A. Yes, the β-subunit of FSH is identical to that of hCG B. No, the test would be specific for the β-subunit of hCG C. Yes, a cross reaction would occur because of structural similarities D. No, the structure of FSH and hCG are not at all similar

B Luteinizing hormone, FSH, and hCG share a common α-subunit but have different β subunits. A test for hCG using a monoclonal antibody would be specific for hCG provided that the antibody was directed against an antigenic determinant on the carboxy terminal end of the β subunit.

Serological tests for which disease may give a false- positive result if the patient has Lyme disease? A. AIDS B. Syphilis C. Cold agglutinins D. Hepatitis C

B Lyme disease is caused by a spirochete and may give positive results with some specific treponemal antibody tests for syphilis.

Which of the following is used in rapid slide tests for detection of rheumatoid factors? A. Whole IgM molecules B. Fc portion of the IgG molecule C. Fab portion of the IgG molecule D. Fc portion of the IgM molecule

B Rheumatoid factors react with the Fc portion of the IgG molecule and are usually IgM. This is the basis of rapid agglutination tests for RA. Particles of latex or cells are coated with IgG. Addition of serum containing rheumatoid factor results in visible agglutination.

SITUATION: A 54-year-old man was admitted to the hospital after having a seizure. Many laboratory tests were performed, including an RPR, but none of the results were positive. The physician suspects a case of late (tertiary) syphilis. Which test should be performed next? A. Repeat RPR, then perform VDRL B. Treponemal test such as MHA-TP on serum C. VDRL on CSF D. No laboratory test is positive for late (tertiary) syphilis

B Serum antibody tests such as RPR and VDRL are often negative in cases of late syphilis. However, treponemal tests remain positive in over 95% of cases. The VDRL test on CSF is the most specific test for diagnosis of neurosyphilis because treponemal tests remain positive after treatment. It should be used as the confirmatory test when the serum treponemal test is positive. However, the CSF VDRL is limited in sensitivity and would not be positive if the serum MHA-TP or FTA-ABS was negative.

Why is skin testing the most widely used method to test for a type I hypersensitivity reaction? A. It causes less trauma and is more cost effective than other methods B. It has greater sensitivity than in vitro measurements C. It is more likely to be positive for IgE-specific allergens than other methods D. It may be used to predict the development of further allergen sensitivity

B Skin testing is considered much more sensitive than in vitro tests that measure either total or antigen-specific IgE.

What immune elements are involved in a positive skin test for tuberculosis? A. IgE antibodies B. T cells and macrophages C. NK cells and IgG antibody D. B cells and IgM antibody

B T cells and macrophages are the immune elements primarily responsible for the clinical manifestations of a positive tuberculosis test. Reactions usually take 72 hours to reach peak development and are characteristic of localized type IV cell-mediated hypersensitivity. The skin reaction is characterized by a lesion containing a mononuclear cell infiltrate.

Which of the following fungal organisms is best diagnosed by an antibody detection test as opposed to an antibody detection assay? A. Histoplasma B. Cryptococcus C. Candida D. Aspergillus

B The Cryptococcus antibody response is not a reliable indicator of a current infection; thus, an antigen assay is normally used to monitor the disease. The antigen assay may be used for serum or spinal fluid and will decline in response to treatment much faster than a traditional antibody test. A urinary antigen test is available for histoplasmosis, and a serum galactomannan assay is available for Aspergillus. Those two assays preform better than antibody detection. No antigen test is available for Candida, thus antibody is the best serologic procedure for this organism.

Which test would measure the coating of red cells by antibody as occurs in hemolytic transfusion reactions? A. Indirect antiglobulin test (IAT) B. Direct antiglobulin test (DAT) C. ELISA D. Hemagglutination

B The DAT test measures antibody that has already coated RBCs in vivo. Direct antiglobulin and direct immunofluorescence tests use anti-immunoglobulin to detect antibody-sensitized cells.

Which test detects antibodies that have attached to tissues, resulting in a type-II cytotoxic reaction? A. Migration inhibition factor assay (MIF) B. Direct immunofluorescence (IF) C. Immunofixation electrophoresis (IFE) D. Hemagglutination

B The direct IF test detects the presence of antibody that may cause a type II cytotoxic reaction. For example, renal biopsies from patients with Goodpasture's syndrome exhibit a smooth pattern of fluorescence along the basement membrane after reaction with fluorescein isothiocyanate (FITC) conjugated anti-immunoglobulin. The reaction detects antibodies against the basement membrane of the glomeruli.

What is the main advantage of the recovery and reinfusion of autologous stem cells? A. It slows the rate of rejection of transplanted cells B. It prevents graft-versus-host disease C. No HLA testing is required D. Engraftment occurs in a more efficient sequence

B The main advantage to the patient from the reinfusion of autologous stem cells is that the procedure prevents graft-versus-host disease, especially in the immunocompromised patient. Although HLA testing is not required, this is not the primary advantage for patient care.

Interpret the following microcytotoxicity results: A9 and B12 cells damaged; A1 and Aw19 cells intact. A. Positive for A1 and Aw19; negative for A9 and B12 B. Negative for A1 and Aw19; positive for A9 and B12 C. Error in test system; retest D. Impossible to determine

B The microcytotoxicity test is based upon the reaction of specific antisera and HLA antigens on test cells. Cells damaged by the binding of antibody and complement are detected with a supravital dye such as eosin.

A patient presents with clinical symptoms of celiac disease. Tests for anti-tissue transglutaminase and antigliadin antibodies are negative. Which of the following tests should be ordered? A. IgG level B. HLA DQ typing C. HLA DR typing D. IgM level

B While antibodies to tissue transglutaminase and gliadin are often found in celiac disease, their combined sensitivity is less than 100%. Celiac disease is almost exclusively associated with the presence of HLA DQ2 and/or HLA DQ8. These HLA genes are not diagnostic of celiac disease, but provide a testing alternative in antibody-negative individuals who meet the clinical diagnostic criteria for celiac disease.

Which increase in antibody titer (dilution) best indicates an acute infection? A. From 1:2 to 1:8 B. From 1:4 to 1:16 C. From 1:16 to 1:256 D. From 1:64 to 1:128

C A fourfold (2 tube) or greater increase in antibody titer is usually indicative of an acute infection. Although answers A and B show a fourfold rise in titer, answer C shows a 16-fold rise in titer and is the most definitive. In most serological tests, a single high titer is insufficient evidence of acute infection unless specific IgM antibodies are measured because age, individual variation, immunologic status, and history of previous exposure (or vaccination) cause a wide variation in normal serum antibody titers.

An immunosuppressed patient has an unexplained anemia. The physician suspects a parvovirus B19 infection. A parvovirus IgM test is negative. The next course of action is to tell the physician: A. The patient does not have parvovirus B. A convalescent specimen is recommended in 4 weeks to determine if a fourfold rise in titer has occurred C. A parvovirus PCR is recommended D. That a recent transfusion for the patient's anemia may have resulted in a false-negative assay and the patient should be retested in 4 weeks

C A negative IgM assay rarely rules out an infection. While a convalescent specimen may be useful in many cases, in an immunosuppressed patient the convalescent specimen may remain negative in the presence of an infection. Thus a parvovirus PCR test is the preferred choice in this case. A false-negative result could conceivably be caused by multiple whole blood or plasma transfusions, but retesting for antibody a month later would not be beneficial to the patient.

Which immunofluorescence pattern indicates the need for ENA testing by Ouchterlony immunodiffusion, Multiplex, or ELISA assays? A. Homogeneous or solid B. Peripheral or rim C. Speckled D. Nucleolar

C A speckled pattern is often due to the presence of antibodies against the extractable nuclear antigens, such as Sm, RNP, SSA, and SSB. Homogenous and rim patterns suggest antibodies to double-stranded DNA. The homogeneous pattern may also be seen with antibodies to deoxyribonuclear protein, which is not an ENA. Nucleolar patterns often indicate antibodies to RNA or fibrillarin.

Which method is the most sensitive for quantitation of AFP? A. Double immunodiffusion B. Electrophoresis C. Enzyme immunoassay D. Particle agglutination

C AFP is a glycoprotein that is produced in about 80%-90% of patients with hepatoma and in a lower percentage of patients with other tumors, including retinoblastoma, breast, uterine, and pancreatic cancer. The upper reference limit for serum is only 10 ng/mL, which requires a sensitive method of assay such as EIA. The high analytical sensitivity of immunoassays permits detection of reduced AFP levels in maternal serum associated with Down syndrome, as well as elevated levels associated with spina bifida.

A woman who has been pregnant for 12 weeks is tested for toxoplasmosis. Her IgM ELISA titer is 2.6 (reference range < 1.6), and her IgG ELISA value is 66 (reference range < 8). The physician asks you if these results indicated an infection during the past 12 weeks. Which of the following tests would you recommend to determine if the woman was infected during her pregnancy? A. Toxo PCR on amniotic fluid B. Toxo IgM on amniotic fluid C. Toxo IgG avidity D. Amniotic fluid culture

C Although IgM is positive, in toxoplasmosis, specific IgM may remain detectable for a year or more following infection. IgG avidity, or the strength of binding of a serum to the antigen of interest, is a useful method to determine if an infection is recent or in the distant past. IgG avidity will increase with time following an infection. Amniotic fluid testing is not useful for determining when the mother might have been infected.

Which of the following symptoms in a young child may indicate an immunodeficiency syndrome? A. Anaphylactic reactions B. Severe rashes and myalgia C. Recurrent bacterial, fungal, and viral infections D. Weight loss, rapid heartbeat, breathlessness

C An immunodeficiency syndrome should be considered in a young child who has a history of recurrent bacterial, fungal, and viral infections manifested after the disappearance of maternal IgG. Immunodeficiency disorders may involve deficiencies in production and/or function of lymphocytes and phagocytic cells or a deficiency in production of a complement factor. Choice of laboratory tests is based upon the patient's clinical presentation, age, and history.

An antinuclear antibody test is performed on a specimen from a 55-year-old woman who has unexplained joint pain. The IFA result is a titer of 40 and a homogeneous pattern. The appropriate follow-up for this patient is: A. Anti-DNA assay B. Extractable nuclear antigen (ENA) testing C. Retest ANA in 3-6 months D. CH50 complement assay

C Approximately 25% of women in this age range may have low titer-positive ANA assays with no demonstrable connective tissue disease. A patient with anti-DNA-positive SLE would be expected to have a much higher titer (> 160) in an IFA assay. A similar titer would be expected for an ENA positive specimen, although the pattern would be speckled. Complement testing would not be indicated with this low titer in a 55-year-old female.

All positive cold agglutinin tubes remain positive after 37°C incubation except the positive control. What is the most likely explanation for this situation? A. High titer cold agglutinins B. Contamination of the test system C. Antibody other than cold agglutinins D. Faulty water bath

C Cold agglutinins do not remain reactive above 30°C, and agglutination must disperse following incubation at 37°C. The most likely explanation when agglutination remains after 37°C incubation is that a warm alloantibody or autoantibody is present.

Interpret the following description of an immunofixation electrophoresis assay of urine. Dense wide bands in both the κ and λ lanes. No bands present in the heavy-chain lanes. A. Normal B. Light chain disease C. Increased polyclonal Fab fragments D. Multiple myeloma

C Heavy wide bands seen with both anti-κ and anti-λ antisera indicate excessive light-chain excretion. Light-chain disease would show a heavy restricted band for one of the light-chain reactions, but not both. The finding of excess λ and κ chains indicates a polyclonal gammopathy with increased immunoglobulin turnover and excretion of the light chains as Fab fragments.

A dialysis patient is positive for both hepatitis B surface antigen and hepatitis B surface antibody. The physician suspects a laboratory error. Do you agree? A. Yes; the patient should not test positive for both HBsAg and HBsAb B. No; incomplete dialysis of a patient in the core window phase of hepatitis B infection will yield this result C. No; it is likely the patient has recently received a hepatitis B booster vaccination and could have these results D. Perhaps; a new specimen should be submitted to clear up the confusion

C Hepatitis B surface antigen will remain detectable at low levels following a vaccination for up to 1-2 weeks. Thus, patients who have received a second injection of hepatitis B vaccine may have anti-hepatitis B surface antigen and detectable antigen for a brief period of time. This has been reported more frequently in dialysis and pediatric populations.

What disease is indicated by a high titer of anti-Sm (anti-Smith) antibody? A. Mixed connective tissue disease (MCTD) B. RA C. SLE D. Scleroderma

C High titer anti-Sm is indicative of SLE. Anti-Sm is one of two antibodies against saline extractable nuclear antigens, the other being anti-RNP. These antibodies cause a speckled pattern of immunofluorescence.

Which of the following tests is used to detect circulating immune complexes in the serum of some patients with systemic autoimmune diseases such as rheumatoid arthritis? A. Direct immunofluorescence B. Enzyme immunoassay C. Assay of cryoglobulins D. Indirect antiglobulin test

C Most autoimmune diseases involve the formation of antigen-antibody complexes that deposit in the tissues, causing local inflammation and necrosis induced by complement activation, phagocytosis, WBC infiltration, and lysosomal damage. Some patients make monoclonal or polyclonal antibodies with rheumatoid factor activity that bind to serum immunoglobulins, forming aggregates that are insoluble at 4°C. These circulating immune complexes are detected by allowing a blood sample to clot at 37°C, transferring the serum to a sedimentation rate tube, and then incubating the serum at 4°C for 3 days.

Free monoclonal light chains are often present in the serum of multiple myeloma patients, and may be useful for disease monitoring. Which of the following assays would be recommended to detect the presence of serum-free light chains? A. Serum protein electrophoresis B. Urine immunofixation C. Nephelometry D. ELISA

C Serum-free light chains are a sensitive indicator of a monoclonal gammopathy. They are often not present in sufficient quantity to show a band on a protein electrophoresis gel. Detecting light chains in the urine is not an indicator of what the serum levels may be. Serum immunoglobulin heavy and light chains are most commonly measured by rate or endpoint nephelometry. ELISA assays are most often used to measure specific antibody levels, not to quantitate immunoglobulin heavy- or light-chain isotypes.

What is measured in the CH50 assay? A. RBC quantity needed to agglutinate 50% of antibody B. Complement needed to lyse 50% of RBCs C. Complement needed to lyse 50% of antibody- sensitized RBCs D. Antibody and complement needed to sensitize 50% of RBCs

C The CH50 is the amount of complement needed to lyse 50% of standardized hemolysin-sensitized sheep RBCs. It is expressed as the reciprocal of the serum dilution resulting in 50% hemolysis. Low levels are associated with deficiency of some complement components and active systemic autoimmune diseases in which complement is being consumed.

Which in vitro test measures IgE levels against a specific allergen? A. Histamine release assay B. Radioimmunosorbent test (RIST) C. Fluorescent allergosorbent test (FAST) D. Precipitin radioimmunosorbent test (PRIST)

C The FAST is a fluorescent assay that measures specific IgE; the RIST and PRIST tests are radioimmunoassays that measure total IgE. The FAST procedure has replaced the RAST, or radioallergosorbent assay. The histamine release assay measures the amount of histamine. Allergen-specific IgE assays are available based upon solid-phase enzyme immunoassay. The allergen is covalently bound to a cellulose solid phase and reacts with specific IgE in the serum. After washing, enzyme (β-galactosidase)-labeled monoclonal anti-IgE is added. The unbound antibody conjugate is washed away and fluorogenic substrate (4-methylumbelliferyl-β-D-galactose) is added. Fluorescence is directly proportional to specific IgE.

In monitoring an HIV-infected patient, which parameter may be expected to be the most sensitive indicator of the effectiveness of antiretroviral treatment? A. HIV antibody titer B. CD4:CD8 ratio C. HIV viral load D. Absolute total T-cell count

C The HIV viral load will rise or fall in response to treatment more quickly than any of the other listed parameters. The absolute CD4 count is also an indicator of treatment effectiveness and is used in resource-poor areas that might not have facilities for molecular testing available. Note that the absolute CD4 count is not one of the choices, however.

SITUATION: A patient with RA has acute pneumonia but a negative throat culture. The physician suspects an infection with Mycoplasma pneumoniae and requests an IgM-specific antibody test. The test is performed directly on serial dilutions of serum less than 4 hours old. The result is positive, giving a titer of 1:32. However, the test is repeated 3 weeks later, and the titer remains at 1:32. What best explains these results? A. IgM-specific antibodies do not increase fourfold between acute and convalescent serum B. The results are not significant because the initial titer was not accompanied by a positive test for cold agglutinins C. Rheumatoid factor caused a false-positive test result D. Insufficient time had elapsed between measurement of acute and convalescent samples

C The IgM-specific antibody test for M. pneumoniae detects antibodies to mycoplasmal membrane antigens and, unlike cold agglutinins, is specific for M. pneumoniae. A positive result (titer of 1:32 or higher) occurs during the acute phase in about 87% of M. pneumoniae infections and does not need to be confirmed by assay of convalescent serum. However, patients with RA may show a false-positive reaction because rheumatoid factor in their serum can react with the conjugated anti-IgM used in the test. For this reason, serum from patients known or suspected to have rheumatoid factor (RF) must be pretreated. The serum is heated to 56°C to aggregate the RF, and the aggregated immunoglobulin is removed by a chromatography minicolumn.

An initial and repeat ELISA test for antibodies to HIV-1 are both positive. A Western blot shows a single band at gp160. The patient shows no clinical signs of HIV infection, and the patient's CD4 T-cell count is normal. Based upon these results, which conclusion is correct? A. Patient is diagnosed as HIV-1-positive B. Patient is diagnosed as HIV-2-positive C. Results are inconclusive D. Patient is diagnosed as HIV-1-negative

C The Western blot test is used as a confirmatory test for HIV, but it is not as sensitive as enzyme immunoassay tests using polyvalent HIV antigens derived from cloned HIV genes. The Western blot test is considered positive only if antibodies to at least two of three viral antigens—p24, gp41, and gp160/120—are detected. The presence of a single band is indeterminate. Over the course of the next 3 months, two or more antibodies will be detected if the patient is HIV positive; however, antibodies to a single viral protein may be caused by a cross reaction, and this patient may fail to seroconvert. This result should be reported as indeterminate, and the patient should be retested in 3 months. Alternatively, a more sensitive confirmatory test such as PCR or immunofluorescence may be performed.

Inflammation involves a variety of biochemical and cellular mediators. Which of the following may be increased within 72 hours after an initial infection? A. Neutrophils, macrophages, antibody, complement, α1-antitrypsin B. Macrophages, T cells, antibody, haptoglobin, fibrinogen C. Neutrophils, macrophages, complement, fibrinogen, C-reactive protein D. Macrophages, T cells, B cells, ceruloplasmin, complement

C The correct list, in which all mediators are involved in an inflammatory response within 72 hours after initial infection, is neutrophils, macrophages, complement, fibrinogen, and C-reactive protein. Phagocytic cells, acute phase reactants, and fibrinolytic factors enter the site of inflammation. Antibody and lymphocytes do not enter until later.

A patient received 2 units of RBCs following surgery. Two weeks after the surgery, the patient was seen by his physician and exhibited mild jaundice and slightly elevated liver enzymes. Hepatitis testing, however, was negative. What should be done next? A. Nothing until more severe or definitive clinical signs develop B. Repeat hepatitis testing immediately C. Repeat hepatitis testing in a few weeks D. Check blood bank donor records and contact donor(s) of transfused units

C The level of HBsAg may not have reached detectable levels, and antibodies to HBc and HCV would not have yet developed. Waiting 1 or 2 weeks and repeating the tests may reveal evidence of hepatitis virus infection.

A patient received 5 units of fresh frozen plasma (FFP) and developed a severe anaphylactic reaction. He has a history of respiratory and gastrointestinal infections. Post-transfusion studies showed all 5 units to be ABO-compatible. What immunologic test would help to determine the cause of this transfusion reaction? A. Complement levels, particularly C3 and C4 B. Flow cytometry for T-cell counts C. Measurement of immunoglobulins D. NBT test for phagocytic function

C The patient had an anaphylactic reaction to a plasma product. This, combined with the history of respiratory and gastrointestinal infections, suggests a selective IgA deficiency. Measurement of immunoglobulins would be helpful in this case. A low serum IgA and normal IgG substantiate the diagnosis of selective IgA deficiency. Such patients frequently produce anti-IgA, which is often responsible for a severe transfusion reaction when ABO-compatible plasma is administered.

Which disease is likely to show a rim (peripheral) pattern in an immunofluorescence (IF) microscopy test for ANA? A. Mixed connective tissue disease (MCTD) B. Rheumatoid arthritis C. Systemic lupus erythematosus D. Scleroderma

C The rim or peripheral pattern seen in indirect immunofluorescence techniques is most commonly found in cases of active SLE. The responsible autoantibody is highly correlated to anti-double-stranded DNA (anti-dsDNA).

Which disease may be expected to show an IgM spike on an electrophoretic pattern? A. Hypogammaglobulinemia B. Multicystic kidney disease C. Waldenström's macroglobulinemia D. Wiskott-Aldrich syndrome

C Waldenström's macroglobulinemia is a malignancy of plasmacytoid lymphocytes involving both the bone marrow and lymph nodes. The malignant cells secrete monoclonal IgM and are in transition from B cells to plasma cells. In contrast to multiple myeloma, osteolytic bone lesions are not found.

A patient has a prostate-specific antigen level of 60 ng/mL the day before surgery to remove a localized prostate tumor. One week following surgery, the serum PSA was determined to be 8 ng/mL by the same method. What is the most likely cause of these results? A. Incomplete removal of the malignancy B. Cross reactivity of the antibody with another tumor antigen C. Testing too soon after surgery D. Hook effect with the PSA assay

C When monitoring the level of a tumor marker for treatment efficacy or recurrence, the half-life of the protein must be considered when determining the testing interval. PSA has a half-life of almost 4 days and would not reach normal levels after surgery for approximately 3-4 weeks. The hook effect is the result of very high antigen levels giving a lower than expected result in a double antibody sandwich assay when both antibodies and sample are added at the same time.

A transplant patient began to show signs of rejection 8 days after receipt of the transplanted organ, and the organ was removed. What immune elements might be found in the rejected organ? A. Antibody and complement B. Primarily antibody C. Macrophages D. T cells

D Acute rejection occurs within 3 weeks of transplantation. The immune element most likely to be involved in an acute rejection is the T cell in a type IV, delayed hypersensitivity (cell-mediated) reaction. Preformed antibody, and possibly complement, is usually involved in hyperacute (immediate) rejection and chronic rejection.

Which of the following serial dilutions contains an incorrect factor? A. 1:4, 1:8, 1:16 B. 1:1, 1:2, 1:4 C. 1:5, 1:15, 1:45 D. 1:2, 1:6, 1:12

D All the dilutions are multiplied by the same factor in a progression except the last one: 1:2 to 1:6 is × 3, whereas 1:6 to 1:12 is × 2. Threefold dilutions of a 1:2 dilution would result in a 1:6 followed by a 1:18.

What antibodies are represented by the nucleolar pattern in the immunofluorescence test for antinuclear antibodies? A. Antihistone antibodies B. Anti-dsDNA antibodies C. Anti-ENA (anti-Sm and anti-RNP) antibodies D. Anti-RNA antibodies

D Anti-RNA antibodies are represented by the nucleolar pattern. This pattern may be seen in most systemic autoimmune diseases and is especially common in patients with scleroderma. Anti-RNA and anti-Sm are not usually found in patients with mixed connective tissue disease. This is a syndrome involving aspects of SLE, RA, scleroderma, and polymyositis. The immunofluorescence pattern most often seen in MCTD is the speckled pattern caused by anti-RNP.

A renal transplant recipient is found to have a rising creatinine level and reduced urine output. The physician orders a "Urine PCR" assay. When you call to find out what organism the physician wants to identify, you are told: A. Hepatitis C virus B. Legionella pneumophila C. EBV D. BK virus

D BK virus is a polyoma virus that can cause renal and urinary tract infections. The virus is an opportunistic pathogen and has become a well-recognized cause of poor renal function in kidney transplant recipients. Antibody testing is not practical or useful for this infection. The principal diagnostic assays are urinary cytology, and specific BK virus PCR testing in urine and serum. Although Legionella pneumophila can be diagnosed through a urinary antigen assay, that organism is not a primary cause of renal insufficiency in transplant patients.

Your cytology laboratory refers a Papanicolaou smear specimen to you for an assay designed to detect the presence of a virus associated with cervical cancer. You perform: A. An ELISA assay for anti-HSV-2 antibodies B. A molecular assay for HSV-2 C. An ELISA assay for HPV antibodies D. A molecular assay for HPV

D Cervical cell atypia and cervical cancer are associated with specific high-risk serotypes of human papilloma virus (HPV) infections. Although HPV antibody assays are available, they are not serotype specific, nor do they relate to disease activity. Thus molecular probe assays are the tests of choice to detect high-risk HPV infection. Although HSV-2 is associated with genital herpesvirus, that virus has not been shown to cause cervical cancer.

All tubes (dilutions) except the negative control are positive for cold agglutinins. This indicates: A. Contaminated red cells B. A rare antibody against red cell antigens C. The sample was stored at 4°C prior to separating serum and cells D. Further serial dilution is necessary

D Cold agglutinins may be measured in patients who have cold agglutinin disease, a cold autoimmune hemolytic anemia. In such cases, titers can be as high as 106. If all tubes (dilutions) for cold agglutinins are positive, except the negative control, then a high titer of cold agglutinins is present in the sample. Further serial dilutions should be performed.

In testing for DiGeorge's syndrome, what type of laboratory analysis would be most helpful in determining the number of mature T cells? A. Complete blood count B. Nitroblue tetrazolium (NBT) test C. T-cell enzyme assays D. Flow cytometry

D DiGeorge's syndrome is caused by a developmental failure or hypoplasia of the thymus, and results in a deficiency of T lymphocytes and cell-mediated immune function. The T-cell count is low, but the level of immunoglobulins is usually normal. Flow cytometry is most helpful in determining numbers and subpopulations of T cells.

What is the main use of laboratory tests to detect antibodies to islet cells and insulin in cases of insulin-dependent diabetes mellitus (IDDM)? A. To regulate levels of injected insulin B. To diagnose IDDM C. To rule out the presence of other autoimmune diseases D. To screen susceptible individuals prior to destruction of β cells

D Fasting hyperglycemia is the primary finding used to diagnose IDDM. For individuals with an inherited susceptibility to the development of IDDM, laboratory tests for the detection of antibodies to islet cells and insulin may help to initiate early treatment before complete destruction of β cells.

A hospital employee received the final dose of the hepatitis B vaccine 3 weeks ago. She wants to donate blood. Which of the following results are expected from the hepatitis screen, and will she be allowed to donate blood? A. HBsAg, positive; anti-HBc, negative—she may donate B. HBsAg, negative; anti-HBc, positive—she may not donate C. HBsAg, positive; anti-HBc, positive—she may not donate D. HBsAg, negative; anti-HBc, negative—she may donate

D She may donate if she is symptom free. The response to hepatitis B vaccine would include a positive result for anti-HBs, a test not normally a part of routine donor testing. She will be negative for HBsAg and anti-HBc.

You are evaluating an ELISA assay as a replacement for your immunofluorescent antinuclear antibody test. You test 50 specimens in duplicate on each assay. The ELISA assay uses a HEp-2 extract as its antigen source. The correlation between the ELISA and the IFA tests is only 60% (30 of 50 specimens agree). Which of the following is the next best course of action? A. Test another 50 specimens B. Perform a competency check on the technologists who performed the tests C. Order a new lot of both kits and then retest on the new lots D. Refer the discrepant specimens for testing by another method

D In this situation, you have already tested the specimens in duplicate. Testing an additional 50 specimens will not change the fact that you have 20 discrepant specimens. The best course of action is to determine what antibodies are actually present in these specimens. Then, you can determine whether the ELISA or IFA is a better procedure for detecting the most clinically relevant antibodies. You could perform clinical chart reviews as an alternative, but obtaining that data would be difficult and much of it may be subjective.

An IFE revealed excessive amounts of polyclonal IgM and low concentrations of IgG and IgA. What is the most likely explanation of these findings and the best course of action? A. Proper amounts of antisera were not added; repeat both tests B. Test specimen was not added properly; repeat both procedures C. Patient has common variable immunodeficiency; perform B-cell count D. Patient has immunodeficiency with hyper-M; perform immunoglobulin levels

D Low plasma concentrations of IgG and IgA and an abundance of IgM is consistent with hyper-IgM syndrome. Most cases are X-linked and result from a mutation of the gene TNFSF5 that encodes a receptorneeded for switching immunoglobulin production. Patients with common variable immunodeficiency have low serum IgG, IgA, and IgM.

A patient with symptoms associated with SLE and scleroderma was evaluated by immunofluorescence microscopy for ANAs using the HEp-2 cell line as substrate. The cell line displayed a mixed pattern of fluorescence that could not be separated by serial dilutions of the serum. Which procedure would be most helpful in determining the antibody profile of this patient? A. Use of a different tissue substrate B. Absorption of the serum using the appropriate tissue extract C. Ouchterlony technique D. ELISA tests for specific antibodies

D Many patients with multiorgan autoimmune disease display symptoms that overlap two or more diseases and have complex mixtures of serum autoantibodies. The HEp-2 substrate is the most sensitive cell line for immunofluorescent microscopy because it contains cells in various mitotic stages, which exposes the serum to more antigens. Use of a nonhuman substrate such as Crithidia may help to identify dsDNA antibodies but would not aid in differentiating all of the antibodies in a complex mixture. Ouchterlony immunodiffusion helps to identify specific ANAs but has limited sensitivity. The best method is ELISA because it is more sensitive than immunofluorescence microscopy and can quantitate antibodies to specific antigens. ELISA is often used to measure antibodies to extractable nuclear antigens, which may be partially or completely lost during fixation of cells used for immunofluorescent microscopy. These antibodies cause a speckled pattern and are seen in a wide range of autoimmune diseases. Identification of the anti ENA specificities is helpful in differentiating these diseases.

All of the following tests may be abnormal in a type III immune complex reaction except: A. C1q-binding assay by ELISA B. Raji cell assay C. CH50 level D. Mitogen response

D Mitogen stimulation is used to measure T-cell, B-cell, and null-cell responsiveness, which is important in patients displaying anergy and other signs of immunodeficiency. The C1q assay and the Raji cell assays detect circulating immune complexes that are present during a type III reaction. The CH50 level is usually decreased owing to complement activation by the immune complexes. Raji cells are derived from a malignant B-cell line that demonstrates C3 receptors but no surface membrane immunoglobulin. Immune complexes that have fixed complement will bind to Raji cells and can be identified using radiolabeled or enzyme labeled anti-immunoglobulin. More recently, a C3 binding ELISA assay has replaced the Raji cell procedure.

What method may be used for tissue typing instead of serological HLA typing? A. PCR B. Southern blotting C. RFLP D. All of these options

D PCR, Southern blotting, and testing for RFLPs may all be used to identify HLA genes. Many laboratories use PCR technology for the routine determination of HLA type.

Which of the following methods is least likely to give a definitive result for the diagnosis of RA? A. Nephelometric measurement of anti-IgG B. Agglutination testing for rheumatoid factor C. Anti CCP D. Immunofluorescence testing for antinuclear antibodies

D Patients with RA often show a homogeneous pattern of fluorescence in tests for antinuclear antibodies. However, this pattern is seen in a wide range of systemic autoimmune diseases and in many normal persons at a titer below 10. The first two methods listed may be used to identify anti-IgG, which is required to establish a diagnosis of RA Anti CCP is a specific assay for rheumatoid arthritis.

What is the "M" component in monoclonal gammopathies? A. IgM produced in excess B. μ Heavy chain produced in excess C. Malignant proliferation of B cells D. Monoclonal antibody or cell line

D The "M" component refers to any monoclonal protein or cell line produced in a monoclonal gammopathy such as multiple myeloma.

A patient's specimen is strongly positive in an ANA ELISA. Which of the following would not be an appropriate follow up to this result? A. Immunofluorescence test on HEp-2 cells B. Specific ENA ELISA tests C. Specific anti-DNA ELISA D. Rheumatoid factor assay

D The ANA ELISA is a screening assay. A positive result may be followed up by more specific antibody ELISA tests or an ANA immunofluorescence test to determine pattern and titer. The ANA ELISA does not screen for rheumatoid factor.

All of the following tests measure phagocyte function except: A. Leukocyte adhesion molecule analysis B. Di Hydro rhodamine reduction assay C. NBT test D. IL-2 (interleukin-2) assay

D The Di-hydro rhodamine reduction assay and NBT tests are used to diagnose chronic granulomatous disease, an inherited disorder in which phagocytic cells fail to kill microorganisms owing to a defect in peroxide production (respiratory burst). Leukocyte adhesion deficiency is associated with a defect in the production of integrin molecules on the surface of WBCs and their granules. IL-2 is a cytokine produced by activated Th and B cells. It causes B-cell proliferation and increased production of antibody, interferon, and other cytokines. IL-2 can be measured by EIA and is used to detect transplant rejection, which is associated with an increase in the serum and urine levels.

On January 4, a serum protein electrophoresis on a specimen obtained at your hospital in North Dakota from a 58-year-old patient shows a band at the β-γ junction. The specimen was also positive for rheumatoid factor. You recommend that an immunofixation test be performed to determine if the band represents a monoclonal immunoglobulin. Another specimen is obtained 2 weeks later by the physician in his office 30 miles away, and the whole blood is submitted to you for the IFE. The courier placed the whole blood specimen in an ice chest for transport. In this specimen, no β-γ band is seen in the serum protein lane, and the IgM lane is very faint. The rheumatoid factor on this specimen was negative. The physician wants to know what's wrong with your laboratory. Your response is: A. Nothing's wrong with our laboratory; the patient had an infection 2 weeks ago that has cleared up B. Something's wrong with our laboratory—we likely mislabeled one of the specimens; please resubmit a new specimen and we will test it at no charge C. You will run a second specimen using a 2-mercaptoethanol treatment that will eliminate IgM aggregates and allow for more sensitive monoclonal IgM detection D. The physician should redraw another specimen from the patient and this time separate the serum from the clot in his office before sending the specimen in by courier

D The most likely cause of the discrepant results is the presence of a type II cryoglobulin. This is a monoclonal rheumatoid factor. The protein likely precipitated during the courier ride and was thus in the clot when the laboratory separated the serum.

Given a heterophile antibody titer of 224, which of the following results indicate IM? Absorption with Absorption with Beef Guinea Pig Kidney Cells A.Two-tube titer reduction Five-tube titer reduction B.No titer reduction No titer reduction C.Five-tube titer reduction Five-tube titer reduction D.Five-tube titer reduction No titer reduction

A Antibodies to infectious mononucleosis (non- Forssman antibodies) are not neutralized or absorbed by guinea pig antigen (but are absorbed by beef cell antigen). A positive test is indicated by at least a four-tube reduction in the heterophile titer after absorption with beef cells and no more than a three-tube reduction in titer after absorption with guinea pig kidney.

Which hepatitis B markers should be performed on blood products? A. HBsAg and anti-HBc B. Anti-HBs and anti-HBc C. HBeAg and HBcAg D. Anti-HBs and HBeAg

A Blood products are tested for HBsAg, an early indicator of infection, and anti-HBc, a marker that may persist for life. Following recovery from HBV infection, some patients demonstrate negative serology for HBsAg and anti-HBs but are positive for anti-HBc. Such patients are considered infective.

All of the following are immunologic functions of complement except: A. Induction of an antiviral state B. Opsonization C. Chemotaxis D. Anaphylatoxin formation

A Complement components are serum proteins that function in opsonization, chemotaxis, and anaphylatoxin formation but do not induce an antiviral state in target cells. This function is performed by interferons.

Which tests are considered screening tests for HIV? A. ELISA, 4th generation, and rapid antibody tests B. Immunofluorescence, Western blot, radioimmuno-precipitation assay C. Culture, antigen capture assay, DNA amplification D. Reverse transcriptase and messenger RNA (mRNA) assay

A ELISA, rapid antibody tests, as well as the 4th generation automated antigen/antibody combination assays are screening tests for HIV. The 4th generation assays detect both antigen and antibody.

The directions for a slide agglutination test instruct that after mixing the patient's serum and latex particles, the slide must be rotated for 2 minutes. What would happen if the slide were rotated for 10 minutes? A. Possible false-positive result B. Possible false-negative result C. No effect D. Depends on the amount of antibody present in the sample

A Failure to follow directions, as in this case where the reaction was allowed to proceed beyond the recommended time, may result in a false-positive reading. Drying on the slide may lead to a possible erroneous positive reading.

All of the following hepatitis viruses are spread through blood or blood products except: A. Hepatitis A B. Hepatitis B C. Hepatitis C D. Hepatitis D

A Hepatitis A is spread through the fecal-oral route and is the cause of infectious hepatitis. Hepatitis A virus has a shorter incubation period (2-7 weeks) than hepatitis B virus (1-6 months). Epidemics of hepatitis A virus can occur, especially when food and water become , contaminated with raw sewage. Hepatitis E virus is also spread via the oral-fecal route and, like hepatitis A virus, has a short incubation period.

Which hepatitis B marker is the best indicator of early acute infection? A. HBsAg B. HBeAg C. Anti-HBc D. Anti-HBs

A Hepatitis B surface antigen (HBsAg) is the first marker to appear in hepatitis B virus infection. It is usually detected within 4 weeks of exposure (prior to the rise in transaminases) and persists for about 3 months after serum enzyme levels return to normal.

Which immunoglobulin can cross the placenta? A. IgG B. IgM C. IgA D. IgE

A IgG is the only immunoglobulin class that can cross the placenta. All subclasses of IgG can cross the placenta, but IgG2 crosses more slowly. This process requires recognition of the Fc region of the IgG by placental cells. These cells take up the IgG from the maternal blood and secrete it into the fetal blood, providing humoral immunity to the neonate for the first few months after delivery

Which statement best describes immunophenotyping? A. Lineage determination by detecting antigens on the surface of the gated cells using fluorescent antibodies B. Identification of cell maturity using antibodies to detect antigens within the nucleus C. Identification and sorting of cells by front and side-scatter of light from a laser D. Analysis of cells collected by flow cytometry using traditional agglutination reactions

A Immunophenotyping refers to classification of cells (lineage and maturity assignment) using a panel of fluorescent-labeled antibodies directed against specific surface antigens on the cells. Antibodies are referred to by their CD (cluster of differentiation) number. Monoclonal antibodies having a common CD number do not necessarily bind to the same epitope but recognize the same antigen on the cell surface. Reactivity of the selected cells with a panel of antibodies differentiates lymphoid from myeloid cells and identifies the stage of cell maturation.

What has happened in a titer, if tube Nos. 5-7 show a stronger reaction than tube Nos.1-4? A. Prozone reaction B. Postzone reaction C. Equivalence reaction D. Poor technique

A In tubes Nos.1-4, insufficient antigen is present to give a visible reaction because excess antibody has saturated all available antigen sites. After dilution of antibody, tubes Nos.1-4 have the equivalent concentrations of antigen and antibody to allow formation of visible complexes.

Macrophages produce which of the following proteins during antigen processing? A. IL-1 and IL-6 B. γ-Interferon C. IL-4, IL-5, and IL-10 D. Complement components C1 and C3

A Interleukin-1 (IL-1) and IL-6 are proinflammatory macrophage-produced cytokines. In addition to their inflammatory properties, they activate T-helper cells during antigen presentation. γ-Interferon, IL-4, 5, and 10 are all produced by T cells. Complement components are produced by a variety of cells but are not part of the macrophage antigen presentation process.

Which statement concerning non-Forssman heterophile antibody is true? A. It is not absorbed by guinea pig antigen B. It is absorbed by guinea pig antigen C. It does not agglutinate horse RBCs D. It does not agglutinate sheep RBCs

A Non-Forssman antibody is not absorbed by guinea pig antigen. This is one of the principles of the Davidsohn differential test for antibodies to IM. These antibodies are non-Forssman; they are absorbed by sheep, horse, or beef RBCs but not by guinea pig kidney. Therefore, a heterophile titer remaining higher after absorption with guinea pig kidney than with beef RBCs indicates IM.

Which type of nephelometry is used to measure immune complex formation almost immediately after reagent has been added? A. Rate B. Endpoint C. Continuous D. One dimensional

A Rate nephelometry is used to measure formation of small immune complexes as they are formed under conditions of antibody excess. The rate of increase in photodetector output is measured within seconds or minutes and increases with increasing antigen concentration. Antigen concentration is determined by comparing the rate for the sample to that for standards using an algorithm that compensates for nonlinearity. In endpoint nephelometry, reactions are read after equivalence. Immune complexes are of maximal size but may have a tendency to settle out of solution, thereby decreasing the amount of scatter.

A patient was suspected of having a lymphoproliferative disorder. After several laboratory tests were completed, the patient was found to have an IgMκ paraprotein. In what sequence should the laboratory tests leading to this diagnosis have been performed? A. Serum protein electrophoresis (SPE) followed by immunofixation electrophoresis (IFE) B. Immunoglobulin levels followed by SPE C. Total lymphocyte count followed by immunoglobulin levels D. Immunoglobulin levels followed by urine protein electrophoresis

A Serum protein electrophoresis should be performed initially to detect the presence of an abnormal immunoglobulin that demonstrates restricted electrophoretic mobility. A patient producing only monoclonal light chains may not show any abnormal serum finding because the light chains may be excreted in the urine. A positive finding for either serum or urine should be followed by IFE on the positive specimen. This is required to confirm the presence of monoclonal immunoglobulin and to identify the heavy and light chain type.

What comprises the indicator system in an indirect ELISA for detecting antibody? A. Enzyme-conjugated antibody + chromogenic substrate B. Enzyme conjugated antigen + chromogenic substrate C. Enzyme + antigen D. Substrate + antigen

A The ELISA test measures antibody using immobilized reagent antigen. The antigen is fixed to the walls of a tube or bottom of a microtiter well. Serum is added (and incubated) and the antibody binds, if present. After washing, the antigen-antibody complexes are detected by adding an enzyme labeled anti-immunoglobulin. The unbound enzyme label is removed by washing, and the bound enzyme label is detected by adding chromogenic substrate. The enzyme catalyzes the conversion of substrate to colored product.

Which test is most likely to be positive in the tertiary stage of syphilis? A. FTA-ABS B. RPR C. VDRL D. Reagin screen test (RST)

A The FTA-ABS or one of the treponemal tests is more likely to be positive than a nontreponemal test in the tertiary stage of syphilis. In some cases, systemic lesions have subsided by the tertiary stage and the nontreponemal tests become seronegative. Although the FTA-ABS is the most sensitive test for tertiary syphilis, it will be positive in both treated and untreated cases.

Which region of the immunoglobulin molecule can bind antigen? A. Fab B. Fc C. CL D. CH

A The Fab (fragment antigen binding) is the region of the immunoglobulin molecule that can bind antigen. Two Fab fragments are formed from hydrolysis of the immunoglobulin molecule by papain. Each consists of a light chain and the VH and CH1 regions of the heavy chain. The variable regions of the light and heavy chains interact, forming a specific antigen-combining site.

What criteria constitute the classification system for HIV infection? A. CD4-positive T-cell count and clinical symptoms B. Clinical symptoms, condition, duration, and number of positive bands on Western blot C. Presence or absence of lymphadenopathy D. Positive bands on Western blot and CD8-positive T-cell count

A The classification system for HIV infection is based upon a combination of CD4-positive T-cell count (helper T cells) and various categories of clinical symptoms. Classification is important in determining treatment options and the progression of the disease.

Interpret the following quantitative RPR test results. RPR titer: weakly reactive 1:8; reactive 1:8-1:64 A. Excess antibody, prozone effect B. Excess antigen, postzone effect C. Equivalence of antigen and antibody D. Impossible to interpret; testing error

A This patient may be in the secondary stage of syphilis and is producing large amounts of antibody to T. pallidum sufficient to cause antibody excess in the test. The test became strongly reactive only after the antibody was diluted

What type of B cells are formed after antigen stimulation? A. Plasma cells and memory B cells B. Mature B cells C. Antigen-dependent B cells D. Receptor-activated B cells

A. Mature B cells exhibit surface immunoglobulin that may cross link a foreign antigen, thus forming the activated B cell and leading to capping and internalization of antigen. The activated B cell gives rise to plasma cells that produce and secrete immunoglobulins and memory cells that reside in lymphoid organs.

An IFE performed on a serum sample showed a narrow dark band in the lanes containing anti-γ and anti-λ. How should this result be interpreted? A. Abnormally decreased IgG concentration B. Abnormal test result demonstrating monoclonal IgGλ C. Normal test result D. Impossible to determine without densitometric quantitation

B A narrow dark band formed in both the lane containing anti-γ and anti-λ indicates the presence of a monoclonal IgG λ immunoglobulin. A diffuse dark band would indicate a polyclonal increase in IgG that often accompanies chronic inflammatory disorders such as systemic lupus erythematosus (SLE).

Which test, other than serological markers, is most consistently elevated in viral hepatitis? A. Antinuclear antibodies B. Alanine aminotransferase (ALT) C. Absolute lymphocyte count D. Lactate dehydrogenase

B ALT is a liver enzyme and may be increased in hepatic disease. Highest levels occur in acute viral hepatitis, reaching 20-50 times the upper limit of normal.

The interaction between an individual antigen and antibody molecule depends upon several types of bonds such as ionic bonds, hydrogen bonds, hydrophobic bonds, and van der Waals forces. How is the strength of this attraction characterized? A. Avidity B. Affinity C. Reactivity D. Valency

B Affinity refers to the strength of a single antibody- antigen interaction. Avidity is the strength of interactions between many different antibodies in a serum against a particular antigen (i.e., the sum of many affinities).

Given a heterophile antibody titer of 224, which of the following results indicate an error in testing? Absorption with Absorption with Beef Guinea Pig Kidney Cells A.Two-tube titer reduction Five-tube titer reduction B.No titer reduction No titer reduction C.Five-tube titer reduction Five-tube titer reduction D.Five-tube titer reduction No titer reduction

B An individual with a 56 or higher titer in the presumptive test (significant heterophile antibodies) has either Forssman antibodies, non-Forssman antibodies, or both. A testing error has occurred if no reduction in the titer of antibody against sheep RBCs is observed after absorption because absorption should remove one or both types of sheep RBC agglutinins.

Which is the first antibody detected in serum after infection with hepatitis B virus (HBV)? A. Anti-HBs B. Anti-HBc IgM C. Anti-HBe D. All are detectable at the same time

B Antibody to the hepatitis B core antigen (anti-HBc) is the first detectable hepatitis B antibody. It persists in the serum for 1-2 years postinfection and is found in the serum of asymptomatic carriers of HBV. Because levels of total anti-HBc are high after recovery, IgM anti-HBc is a more useful marker for acute infection. Both anti-HBc and anti-HBs can persist for life, but only anti-HBs is considered protective.

How are cytotoxic T cells (Tc cells) and natural killer (NK) cells similar? A. Require antibody to be present B. Effective against virally infected cells C. Recognize antigen in association with HLA class II markers D. Do not bind to infected cells

B Both TC and NK cells are effective against virally infected cells, and neither requires antibody to be present to bind to infected cells. NK cells do not exhibit MHC class restriction, whereas activation of Tc cells require the presence of MHC class I molecules in association with the viral antigen.

What is the purpose of C3a, C4a, and C5a, the split products of the complement cascade? A. To bind with specific membrane receptors of lymphocytes and cause release of cytotoxic substances B. To cause increased vascular permeability, contraction of smooth muscle, and release of histamine from basophils C. To bind with membrane receptors of macrophages to facilitate phagocytosis and the removal of debris and foreign substances D. To regulate and degrade membrane cofactor protein after activation by C3 convertase

B C3a, C4a, and C5a are split products of the complement cascade that participate in various biological functions such as vasodilation and smooth muscles contraction. Theses small pepetides act as anaphylatoxins, e.g., effector molecules that participate in the inflammatory response to assist in the destruction and clearance of foreign antigens.

Which serum antibody response usually characterizes the primary (early) stage of syphilis? A. Antibodies against syphilis are undetectable B. Detected 1-3 weeks after appearance of the primary chancre C. Detected in 50% of cases before the primary chancre disappears D. Detected within 2 weeks after infection

B During the primary stage of syphilis, about 90% of patients develop antibodies between 1 and 3 weeks after the appearance of the primary chancre.

What effect does selecting the wrong gate have on the results when cells are counted by flow cytometry? A. No effect B. Failure to count the desired cell population C. Falsely elevated results D. Impossible to determine

B Gating is the step performed to select the proper cells to be counted. Failure to properly perform this procedure will result in problems in isolating and counting the desired cells. It is impossible to determine if the final result would be falsely elevated or falsely lowered by problems with gating.

Which MHC class of molecule is necessary for antigen recognition by CD4-positive T cells? A. Class I B. Class II C. Class III D. No MHC molecule is necessary for antigen recognition

B Helper T lymphocytes (CD4-positive T cells) recognize antigens only in the context of a class II molecule. Because class II antigens are expressed on macrophages, monocytes, and B cells, the helper T-cell response is mediated by interaction with processed antigen on the surface of these cells.

What would happen if the color reaction phase is prolonged in one tube or well of an ELISA test? A. Result will be falsely decreased B. Result will be falsely increased C. Result will be unaffected D. Impossible to determine

B If the color reaction is not stopped within the time limits specified by the procedure, the enzyme will continue to act on the substrate, producing a falsely elevated test result.

What outcome results from improper washing of a tube or well after adding the enzyme-antibody conjugate in an ELISA system? A. Result will be falsely decreased B. Result will be falsely increased C. Result will be unaffected D. Result is impossible to determine

B If unbound enzyme-conjugated anti-immunoglobulin is not washed away, it will catalyze conversion of substrate to colored product, yielding a falsely elevated result.

Which antibody persists in low-level carriers of hepatitis B virus? A. IgM anti-HBc B. IgG anti-HBc C. IgM anti-HBe D. IgG anti-HBs

B IgG antibodies to the hepatitis B core antigen (anti-HBc) can be detected in carriers who are HBsAg and anti-HBs negative. These persons are presumed infective even though the level of HBsAg is too low to detect. No specific B core IgG test is available, however. This patient would be positive in the anti-B core total antibody assay and negative in the anti-HB core IgM test.

What is the most likely cause when a Western blot or ELISA is positive for all controls and samples? A. Improper pipetting B. Improper washing C. Improper addition of sample D. Improper reading

B Improper washing may not remove unbound enzyme conjugated anti-human globulin, and every sample may appear positive.

An immunofluorescence microscopy assay (IFA) was performed, and a significant antibody titer was reported. Positive and negative controls performed as expected. However, the clinical evaluation of the patient was not consistent with a positive finding. What is the most likely explanation of this situation? A. The clinical condition of the patient changed since the sample was tested B. The pattern of fluorescence was misinterpreted C. The control results were misinterpreted D. The wrong cell line was used for the test

B In an IFA, for example, an antinuclear antibody (ANA) test, the fluorescence pattern must be correlated correctly with the specificity of the antibodies. Both pathological and nonpathological antibodies can occur, and antibodies may be detected at a significant titer in a patient whose disease is inactive. Failure to correctly identify subcellular structures may result in misinterpretation of the antibody specificity, or a false positive caused by nonspecific fluorescence.

What is the most likely explanation when antibody tests for HIV are negative but a polymerase chain reaction test performed 1 week later is positive? A. Probably not HIV infection B. Patient is in the "window phase" before antibody production C. Tests were performed incorrectly D. Clinical signs may be misinterpreted

B In early seroconversion, patients may not be making enough antibodies to be detected by antibody tests. The period between infection with HIV and the appearance of detectable antibodies is called the window phase. Although this period has been reduced to a few weeks by sensitive enzyme immunoassays, patients at high risk or displaying clinical conditions associated with HIV disease should be tested again after waiting several more weeks.

A flow cytometry scattergram of a bone marrow sample shows a dense population of cells located in-between normal lymphoid and normal myeloid cells. What is the most likely explanation? A. The sample was improperly collected B. An abnormal cell population is present C. The laser optics are out of alignment D. The cells are most likely not leukocytes

B Lymphoid cells and myeloid cells display in predictable regions of the scatterplot because of their characteristic size and density. Lymphoid cells cause less forward and side scatter from the laser than do myeloid cells. A dense zone of cells in between those regions is caused by the presence of a large number of abnormal cells, usually blasts. The lineage of the cells can be determined by immunophenotyping with a panel of fluorescent-labeled antibodies.

What substance is detected by the rapid plasma reagin (RPR) and Venereal Disease Research Laboratory (VDRL) tests for syphilis? A. Cardiolipin B. Anticardiolipin antibody C. Anti-T. pallidum antibody D. Treponema pallidum

B Reagin is the name for a nontreponemal antibody that appears in the serum of syphilis-infected persons and is detected by the RPR and VDRL assays. Reagin reacts with cardiolipin, a lipid-rich extract of beef heart and other animal tissues.

A biological false-positive reaction is least likely with which test for syphilis? A. VDRL B. Fluorescent T. pallidum antibody absorption test (FTA-ABS) C. RPR D. All are equally likely to detect a false-positive result

B The FTA-ABS test is more specific for T. pallidum than nontreponemal tests such as the VDRL and RPR and would be least likely to detect a biological false-positive result. The FTA-ABS test uses heat-inactivated serum that has been absorbed with the Reiter strain of T. pallidum to remove nonspecific antibodies. Nontreponemal tests have a biological false-positive rate of 1%-10%, depending upon the patient population tested. False-positive findings are caused commonly by infectious mononucleosis (IM), SLE, viral hepatitis, and human immunodeficiency virus (HIV) infection.

What is the titer in tube No. 8 if tube No. 1 is undiluted and dilutions are doubled? A. 64 B. 128 C. 256 D. 512

B The antibody titer is reciprocal of the highest dilution of serum giving a positive reaction. For doubling dilutions, each tube has one half the amount of serum as the previous tube. Because the first tube was undiluted (neat), the dilution in tube No. 8 is (1/2)7 and the titer equals 27 or 128.

The T-cell antigen receptor is similar to immunoglobulin molecules in that it: A. Remains bound to the cell surface and is never secreted B. Contains V and C regions on each of its chains C. Binds complement D. Can cross the placenta and provide protection to a fetus

B The antigen binding regions of both the α- and β-chains of the T-cell receptor are encoded by V genes that undergo rearrangement similar to that observed in immunoglobulin genes. The α-chain gene consists of V and J segments, similar to an immunoglobulin light chain. The β-chain consists of V, D, and J segments, similar to an immunoglobulin heavy chain. The α- and β-chains each have a single C-region gene encoding the constant region of the molecule. While answer A is true for T-cell receptors, it is not true for immunoglobulins that can be cell bound or secreted. Answers C and D are true for certain immunoglobulin heavy-chain isotypes but are not true for the T-cell receptor.

Which region determines whether an immunoglobulin molecule can fix complement? A. VH B. CH C. VL D. CL

B The composition and structure of the constant region of the heavy chain determine whether that immunoglobulin will fix complement. The Fc fragment (fragment crystallizable) is formed by partial immunoglobulin digestion with papain and includes the CH2 and CH3 domains of both heavy chains. The complement component C1q molecule will bind to the CH2 region of an IgG or IgM molecule.

Which immunoglobulin appears first in the primary immune response? A. IgG B. IgM C. IgA D. IgE characteristics/Immunoglobulins/Functions/1

B The first antibody to appear in the primary immune response to an antigen is IgM. The titer of antiviral IgM (e.g., IgM antibody to cytomegalovirus [anti-CMV]) is more specific for acute or active viral infection than IgG and may be measured to help differentiate active from prior infection.

What is the endpoint for the antistreptolysin O (ASO) latex agglutination assay? A. Highest serum dilution that shows no agglutination B. Highest serum dilution that shows agglutination C. Lowest serum dilution that shows agglutination D. Lowest serum dilution that shows no agglutination

B The latex test for ASO includes latex particles coated with streptolysin O. Serial dilutions are prepared and the highest dilution showing agglutination is the endpoint.

Which control shows the correct result for a valid ASO test? A. SLO control, no hemolysis B. Red cell control, no hemolysis C. Positive control, hemolysis in all tubes D. Hemolysis in both SLO and red cell control

B The red cell control contains no SLO and should show no hemolysis. The SLO control contains no serum and should show complete hemolysis. An ASO titer cannot be determined unless both the RBC and SLO controls demonstrate the expected results.

Interpret the following results for HIV infection. ELISA: positive; repeat ELISA: negative; Western blot: no bands A. Positive for HIV B. Negative for HIV C. Indeterminate D. Further testing needed

B These results are not indicative of an HIV infection and may be due to a testing error in the first ELISA assay. Known false-positive ELISA reactions occur in autoimmune diseases, syphilis, alcoholism, and lymphoproliferative diseases. A sample is considered positive for HIV if it is repeatedly positive by ELISA or other screening method and positive by a confirmatory method.

Toll-like receptors are found on which cells? A. T cells B. Dendritic cells C. B cells D. Large granular lymphocytes

B Toll-like receptors (TLR) are the primary antigen recognition protein of the innate immune system. They are found on antigen-presenting cells such as dendritic cells and macrophages. Eleven TLRs have been described. TLRs recognize certain structural motifs common to infecting organisms. TLR 4, for example, recognizes bacterial lipopolysaccharide (LPS). The name comes from their similarity to the Toll protein in Drosophila.

Tests to identify infection with HIV fall into which three general classification types of tests? A. Tissue culture, antigen, and antibody tests B. Tests for antigens, antibodies, and nucleic acid C. DNA probe, DNA amplification, and Western blot tests D. ELISA, Western blot, and Southern blot tests

B Two common methods for detecting antibodies to HIV are the ELISA and Western blot tests. Two common methods for detecting HIV antigens are ELISA and immunofluorescence. Two common methods for detecting HIV genes are the Southern blot test and DNA amplification using the polymerase chain reaction to detect viral nucleic acid in infected lymphocytes.

Which tests are considered confirmatory tests for HIV? A. ELISA and rapid antibody tests B. Western blot test, HIC-1,2 differentiation assays, and polymerase chain reaction C. Culture, antigen capture assay, polymerase chain reaction D. Reverse transcriptase and mRNA assay

B Western blot, and PCR tests are generally used as confirmatory tests for HIV. An HIV-1,2 differentiation assay is recommended as the confirming procedure following a reactive 4th generation HIV assay. PCR, however, is more often used for early detection of HIV infection, for documenting infant HIV infection, and for following antiviral therapy.

T cells travel from the bone marrow to the thymus for maturation. What is the correct order of the maturation sequence for T cells in the thymus? A. Bone marrow to the cortex; after thymic education, released back to peripheral circulation B. Maturation and selection occur in the cortex; migration to the medulla; release of mature T cells to secondary lymphoid organs C. Storage in either the cortex or medulla; release of T cells into the peripheral circulation D. Activation and selection occur in the medulla; mature T cells are stored in the cortex until activated by antigen

B. Immature T cells travel from the bone marrow to the thymus to mature into functional T cells. Once in the thymus, T cells undergo a selection and maturation sequence that begins in the cortex and moves to the medulla of the thymus. Thymic factors such as thymosin and thymopoietin and cells within the thymus such as macrophages and dendritic cells assist in this sequence. After completion of the maturation cycle, T cells are released to secondary lymphoid organs to await antigen recognition and activation.

Which two organs are considered the primary lymphoid organs in which immunocompetent cells originate and mature? A. Thyroid and Peyer's Patch B. Thymus and bone marrow C. Spleen and mucosal-associated lymphoid tissue (MALT) D. Lymph nodes and thoracic duct

B. The bone marrow and thymus are considered primary lymphoid organs because immunocompetent cells either originate or mature in them. Some immunocompetent cells mature or reside in the bone marrow (the source of all hematopoietic cells) until transported to the thymus, spleen, or Peyer's patches, where they process antigen or manufacture antibody. T lymphocytes, after originating in the bone marrow, travel to the thymus to mature and differentiate.

Which cluster of differentiation (CD) marker appears during the first stage of T-cell development and remains present as an identifying marker for T cells? A. CD 1 B. CD 2 C. CD 3 D. CD 4 or CD 8

B. the CD 2 marker appears during the first stage of T-cell development and can be used to differentiate T cells from other lymphocytes. This T-lymphocyte receptor binds sheep red blood cells (RBCs). This peculiar characteristic was the basis for the classic E rosette test once used to enumerate T cells in peripheral blood. CD2 is not specific for T cells, however, and is also found on large granular lymphocytes (LGL or natural killer [NK]cells).

What type of antigen is used in the RPR card test? A. Live treponemal organisms B. Killed suspension of treponemal organisms C. Cardiolipin D. Tanned sheep cells

C Cardiolipin is extracted from animal tissues, such as beef hearts, and attached to carbon particles. In the presence of reagin, the particles will agglutinate.

Interpret the following results for EBV infection: IgG and IgM antibodies to viral capsid antigen (VCA) are positive. A. Infection in the past B. Infection with a mutual enhancer virus such as HIV C. Current infection D. Impossible to interpret; need more information

C Antibodies to both IgG and IgM VCA are found in a current infection with EBV. The IgG antibody may persist for life, but the IgM anti-VCA disappears within 4 months after the infection resolves.

What is the advantage of 4th-generation rapid HIV tests over earlier rapid HIV tests? A. They use recombinant antigens B. They detect multiple strains of HIV C. They detect p24 antigen D. They are quantitative

C Both 3rd-generation and 4th-generation rapid tests for HIV use recombinant and synthetic HIV antigens conjugated to a solid phase. The multivalent nature of these tests allows for detection of less common subgroups of HIV-1 and simultaneous detection of both HIV-1 and HIV-2. However, the 4th-generation assays also use solid-phase antibodies to p24 antigen to detect its presence. Because p24 antigen appears before antibodies to HIV, 4th-generation tests can detect infection 4-7 days earlier than tests based on antibody detection alone.

Which immunoglobulin(s) help(s) initiate the classic complement pathway? A. IgA and IgD B. IgM only C. IgG and IgM D. IgG only

C Both IgG and IgM are the immunoglobulins that help to initiate the activation of the classic complement pathway. IgM is a more potent complement activator, however.

Which immunoglobulin class(es) has (have) a J chain? A. IgM B. IgE and IgD C. IgM and sIgA D. IgG3 and IgA

C Both IgM and secretory IgA have a J chain joining individual molecules together; the J chain in IgM joins five molecules and the J chain in sIgA joins two molecules.

Blood products are tested for which virus before being transfused to newborns? A. EBV B. Human T-lymphotropic virus II (HTLV-II) C. Cytomegalovirus (CMV) D. Hepatitis D virus

C CMV can be life threatening if transmitted to a newborn through a blood product. HTLV-II is a rare virus, which like HIV, is a T-cell tropic RNA retrovirus. The virus has been associated with hairy cell leukemia, but this is not a consistent finding.

What is the name of the process by which phagocytic cells are attracted to a substance such as a bacterial peptide? A. Diapedesis B. Degranulation C. Chemotaxis D. Pahotaxis

C Chemotaxis is the process by which phagocytic cells are attracted toward an area where they detect a disturbance in the normal functions of body tissues. Products from bacteria and viruses, complement components, coagulation proteins, and cytokines from other immune cells may all act as chemotactic factors.

Which of the following are products of HLA class III genes? A. T-cell immune receptors B. HLA-D antigens on immune cells C. Complement proteins C2, C4, and Factor B D. Immunoglobulin VL regions

C Complement components C2 and C4 of the classic pathway and Factor B of the alternative pathway are class III molecules. HLA-A, HLA-B, and HLA-C antigens are classified as class I antigens, and HLA-D, HLA-DR, HLA-DQ, and HLA-DP antigens as class II antigens.

What is the interpretation when an Ouchterlony plate shows crossed lines between wells 1 and 2 (antigen is placed in the center well and antisera in wells 1 and 2)? A. No reaction between wells 1 and 2 B. Partial identity between wells 1 and 2 C. Nonidentity between wells 1 and 2 D. Identity between wells 1 and 2

C Crossed lines indicate nonidentity between wells 1 and 2. The antibody from well 1 recognizes a different antigenic determinant than the antibody from well 2.

All of the following are functions of immunoglobulins except: A. Neutralizing toxic substances B. Facilitating phagocytosis through opsonization C. Interacting with TC cells to lyse viruses D. Combining with complement to destroy cellular antigens

C Cytotoxic T cells lyse virally infected cells directly, without requirement for specific antibody. The TC cell is activated by viral antigen that is associated with MHC class I molecules on the surface of the infected cell. The activated TC cell secretes several toxins, such as tumor necrosis factor, which destroy the infected cell and virions.

Which method is used to test for HIV infection in infants who are born to HIV-positive mothers? A. ELISA B. Western blot test C. Polymerase chain reaction D. Viral culture

C ELISA and Western blot primarily reflect the presence of maternal antibody. The PCR uses small amounts of blood and does not rely on the antibody response. PCR amplifies small amounts of viral nucleic acid and can detect less than 200 copies of viral RNA per milliliter of plasma. These qualities make PCR ideal for the testing of infants. Nucleic acid methods for HIV RNA include the Roche Amplicor reverse- transcriptase assay, the branched DNA (bDNA) signal amplification method, and the nucleic acid sequence-based amplification (NASBA) method.

Which of the following statements regarding infection with hepatitis D virus is true? A. Occurs in patients with HIV infection B. Does not progress to chronic hepatitis C. Occurs in patients with hepatitis B D. Is not spread through blood or sexual contact

C Hepatitis D virus is an RNA virus that requires the surface antigen or envelope of the hepatitis B virus for entry into the hepatocyte. Consequently, hepatitis D virus can infect only patients who are coinfected with hepatitis B.

Which part of the radial immunodiffusion (RID) test system contains the antisera? A. Center well B. Outer wells C. Gel D. Antisera may be added to any well

C In an RID test system, for example, one measuring hemopexin concentration, the gel would contain the antihemopexin. A standardized volume of serum containing the antigen is added to each well. Antigen diffuses from the well into the gel and forms a precipitin ring by reaction with antibody. At equivalence, the area of the ring is proportional to antigen concentration.

Which outcome indicates a negative result in a complement fixation test? A. Hemagglutination B. Absence of hemagglutination C. Hemolysis D. Absence of hemolysis

C In complement fixation, hemolysis indicates a negative test result. The absence of hemolysis indicates that complement was fixed in an antigen-antibody reaction and, therefore, that the specific complement binding antibody was present in the patient's serum. Consequently, it was not available to react in the indicator system.

Given a heterophile antibody titer of 224, which of the following results indicate serum sickness? Absorption with Absorption with Beef Guinea Pig Kidney Cells A.Two-tube titer reduction Five-tube titer reduction B.No titer reduction No titer reduction C. Five-tube titer reduction Five-tube titer reduction D.Five-tube titer reduction No titer reduction

C In serum sickness, antibodies are neutralized by both guinea pig kidney and beef cell antigens, and at least a three-tube (eightfold) reduction in titer should occur after absorption with both.

Which specimen is the sample of choice to evaluate latent or tertiary syphilis? A. Serum sample B. Chancre fluid C. CSF D. Joint fluid

C Latent syphilis usually begins after the second year of untreated infection. In some cases, the serological tests become negative. However, if neurosyphilis is present, cerebrospinal fluid serology will be positive and the CSF will display increased protein and pleocytosis characteristic of central nervous system infection.

A woman who has had five pregnancies subsequently tests positive for HIV by Western blot. What is the most likely reason for this result? A. Possible cross-reaction with herpes or EBV antibodies B. Interference from medication C. Cross-reaction with HLA antigens in the antigen preparation D. Possible technical error

C Multiparous women often have HLA antibodies. The Western blot antigens are derived from HIV grown in human cell lines having HLA antigens. A cross reaction with HLA antigen(s) in the Western blot could have occurred.

T regulator cells, responsible for controlling autoimmune antibody production, express which of the following phenotypes? A. CD3, CD4, CD8 B. CD3, CD8, CD25 C. CD3, CD4, CD25 D. CD8, CD25, CD56

C T regulator cells are believed to be the primary immune suppressor cells and express CD3, CD4, and CD25. CD25 is the interleukin 2 receptor. CD25 may be expressed by activated T cells, but is constitutively expressed by the T-regulator cells. CD25 expression on T-regulator cells occurs in the thymus and is regulated by the FOXP3 protein.

What is the most likely explanation when a patient has clinical signs of viral hepatitis but tests negative for hepatitis A IgM, hepatitis B surface antigen, and hepatitis C Ab? A. Tests were performed improperly B. The patient does not have hepatitis C. The patient may be in the "core window" D. Clinical evaluation was performed improperly

C The patient may be in the "core window," the period of hepatitis B infection when both the surface antigen and surface antibody are undetectable. The IgM anti-hepatitis B core and the anti-hepatitis B core total antibody assays would be the only detectable markers in the serum of a patient in the core window phase of hepatitis B infection.

Why is a chemiluminescent immunoassay (CIA) or enzyme immunoassay (EIA) the method of choice for detection of certain analytes, such as hormones, normally found in low concentrations? A. Because of low cross reactivity B. Because of high specificity C. Because of high sensitivity D. Because test systems may be designed as both competitive and noncompetitive assays

C The sensitivity of EIA methods producing visible color change, and fluorescent and chemiluminescent products approaches nanogram levels of antibody. These methods are easily automated.

What is the main difficulty associated with the development of an HIV vaccine? A. The virus has been difficult to culture; antigen extraction and concentration are extremely laborious B. Human trials cannot be performed C. Different strains of the virus are genetically diverse D. Anti-idiotype antibodies cannot be developed

C Vaccine development has been difficult primarily because of the genetic diversity among different strains of the virus, and new strains are constantly emerging. HIV-1 can be divided into two main subtypes designated M (for main) and O (for outlier). The M group is further divided into 9 subgroups, designated A-J (there is no E subgroup), based upon differences in the nucleotide sequence of the gag gene. Two remaining subtypes are designed N (non M and non O) and P (a subtype related to SIVgor). A vaccine has yet to be developed that is effective for all of the subgroups of HIV-1.

Which T cell expresses the CD8 marker and acts specifically to kill tumor or virally infected cells? A. Helper T B. T suppressor C. T cytotoxic D. T inducer/suppressor

C. T cytotoxic cells recognize antigen in association with major histocompatibility complex (MHC) class I complexes and act against target cells that express foreign antigens. These include viral antigens and the human leukocyte antigens (HLA) that are the target of graft rejection.

What is the most likely interpretation of the following syphilis serological results? RPR: reactive; VDRL: reactive; MHA-TP: nonreactive A. Neurosyphilis B. Secondary syphilis C. Syphilis that has been successfully treated D. Biological false positive

D A positive reaction with nontreponemal antigen and a negative reaction with a treponemal antigen is most likely caused by a biological false-positive nontreponemal test.

A superantigen, such as toxic shock syndrome toxin-1 (TSST-1), bypasses the normal antigen processing stage by binding to and cross linking: A. A portion of an immunoglobulin molecule and complement component C1 B. Toll-like receptors and an MHC class 1 molecule C. A portion of an immunoglobulin and a portion of a T-cell receptor D. A portion of a T-cell receptor and an MHC class II molecule

D A superantigen binds to the V β portion of the T-cell receptor and an MHC class II molecule. This binding can activate T cells without the involvement of an antigen-presenting cell. In some individuals, a single V β protein that recognizes TSST-1 is expressed on up to 10%-20% of T cells. The simultaneous activation of this amount of T cells causes a heavy cytokine release, resulting in the vascular collapse and pathology of toxic shock syndrome.

The detection of precipitation reactions depends on the presence of optimal proportions of antigen and antibody. A patient's sample contains a large amount of antibody, but the reaction in a test system containing antigen is negative. What has happened? A. Performance error B. Low specificity C. A shift in the zone of equivalence D. Prozone phenomenon

D Although performance error and low specificity should be considered, if a test system fails to yield the expected reaction, excessive antibody preventing a precipitation reaction is usually the cause. Prozone occurs when antibody molecules saturate the antigen sites, preventing cross linking of the antigen-antibody complexes by other antibody molecules. Because the antigen and antibody do not react at equivalence, a visible product is not formed, leading to a false-negative result.

Which CD4:CD8 ratio is most likely in a patient with acquired immunodeficiency syndrome (AIDS)? A. 2:1 B. 3:1 C. 2:3 D. 1:2

D An inverted CD4:CD8 ratio (less than 1.0) is a common finding in an AIDS patient. The Centers for Disease Control and Prevention requires a CD4-positive (helper T) cell count of less than 200/μL or 14% in the absence of an AIDS-defining illness (e.g., Pneumocystis carinii pneumonia) in the case surveillance definition of AIDS.

What corrective action should be taken when an indeterminate pattern occurs in an indirect IFA? A. Repeat the test using a larger volume of sample B. Call the physician C. Have another medical laboratory scientist read the slide D. Dilute the sample and retest

D An unexpected pattern may indicate the presence of more than one antibody. Diluting the sample may help to clearly show the antibody specificities, if they are found in different titers. If the pattern is still atypical, a new sample should be collected and the test repeated.

Which hepatitis antibody confers immunity against reinfection with hepatitis B virus? A. Anti-HBc IgM B. Anti-HBc IgG C. Anti-HBe D. Anti-HBs

D Anti-HBs appears later in infection than anti-HBc and is used as a marker for immunity following infection or vaccination rather than for diagnosis of current infection.

Which immunoglobulin cross links mast cells to release histamine? A. IgG B. IgM C. IgA D. IgE

D IgE is the immunoglobulin that cross links with basophils and mast cells. IgE causes the release of such immune response modifiers as histamine and mediates an allergic immune response

Which statement best describes passive agglutination reactions used for serodiagnosis? A. Such agglutination reactions are more rapid because they are a single-step process B. Reactions require the addition of a second antibody C. Passive agglutination reactions require biphasic incubation D. Carrier particles for antigen such as latex particles are used

D Most agglutination tests used in serology employ passive or indirect agglutination where carrier particles are coated with the antigen. The carrier molecule is of sufficient size so that the reaction of the antigen with antibody results in formation of a complex that is more easily visible.

If only anti-HBs is positive, which of the following can be ruled out? A. Hepatitis B virus vaccination B. Distant past infection with hepatitis B virus C. Hepatitis B immune globulin (HBIG) injection D. Chronic hepatitis B virus infection

D Persons with chronic HBV infection show a positive test result for anti-HBc (IgG or total) and HBsAg but not anti-HBs. Patients with active chronic hepatitis have not become immune to the virus.

A laboratory is evaluating an enzyme-linked immunosorbent assay (ELISA) for detecting an antibody to cyclic citrullinated peptide (CCP), which is a marker for rheumatoid arthritis. The laboratory includes serum from healthy volunteers and patients with other connective tissue diseases in the evaluation. These specimens determine which factor of the assay? A. Sensitivity B. Precision C. Bias D. Specificity

D Specificity is defined as a negative result in the absence of the disease. The non-rheumatoid arthritis specimens would be expected to test negative if the assay has high specificity. Precision is the ability of the assay to repeatedly yield the same results on a single specimen. Both bias and sensitivity calculations would include specimens from rheumatoid arthritis specimens. Although those specimens would be included in the evaluation, they are not listed in the question

What molecule on the surface of most T cells recognizes antigen? A. IgT, a four-chain molecule that includes the tau heavy chain B. MHC protein, a two-chain molecule encoded by the HLA region C. CD3, consisting of six different chains D. TcR, consisting of two chains, alpha and beta

D T cells have a membrane bound receptor (T-cell receptor or TcR) that is antigen specific. This two- chain molecule consists of a single α-chain, similar to an immunoglobulin light chain, and a single β-chain, similar to an immunoglobulin heavy chain. Some T cells may express a γ-δ receptor instead of the α-β molecule. There is no τ heavy chain. MHC and CD3 molecules are present on T cells, but they are not the molecules that give antigen specificity to the cell.

Which of the following cell surface molecules is classified as an MHC class II antigen? A. HLA-A B. HLA-B C. HLA-C D. HLA-DR

D The MHC region is located on the short arm of chromosome 6 and codes for antigens expressed on the surface of leukocytes and tissues. The MHC region genes control immune recognition; their products include the antigens that determine transplantation rejection. HLA-DR antigens are expressed on B cells. HLA-DR2, DR3, DR4, and DR5 antigens show linkage with a wide range of autoimmune diseases.

Which of the following is the most sensitive test to detect congenital syphilis? A. VDRL B. RPR C. Microhemagglutinin test for T. pallidum (MHA-TP) D. Polymerase chain reaction (PCR)

D The PCR will amplify a very small amount of DNA from T. pallidum and allow for detection of the organism in the infant. Antibody tests such as VDRL and RPR may detect maternal antibody only, not indicating if the infant has been infected.

Interpret the following results for HIV infection. HIV 1,2 ELISA: positive; HIV-1 Western blot: indeterminate; HIV-1 p24 antigen: negative A. Positive for antibodies to human immunodeficiency virus, HIV-1 B. Positive for antibodies to human immunodeficiency virus, HIV-2 C. Cross reaction; biological false-positive result D. Additional testing required

D The indeterminate Western blot and negative p24 antigen assay indicate that HIV-1 infection is unlikely, However, additional testing is required to determine if the patient has antibodies to HIV-2 or if this could be a false-positive ELISA assay.

A 12-year old girl has symptoms of fatigue and a localized lymphadenopathy. Laboratory tests reveal a peripheral blood lymphocytosis, a positive RPR, and a positive spot test for IM. What test should be performed next? A. HIV test by ELISA B. VDRL C. Epstein-Barr virus (EBV) specific antigen test D. Treponema pallidum particle agglutination (TP-PA) test

D The patient's symptoms are nonspecific and could be attributed to many potential causes. However, the patient's age, lymphocytosis, and serological results point to infectious mononucleosis (IM). The rapid spot test for antibodies seen in IM is highly specific. The EBV-specific antigen test is more sensitive but is unnecessary when the spot test is positive. HIV infection is uncommon at this age and is often associated with generalized lymphadenopathy and a normal or reduced total lymphocyte count. IM antibodies are commonly implicated as a cause of biological false-positive nontreponemal tests for syphilis. Therefore, a treponemal test for syphilis should be performed to document this phenomenon in this case.

Rapid assays for influenza that utilize specimens obtained from nasopharyngeal swabs detect: A. IgM anti-influenza B. IgA anti-influenza C. IgA-influenza Ag immune complexes D. Influenza antigen

D The rapid influenza assays are antigen detection methods. They are designed to detect early infection, before antibody is produced.

Which is most likely a positive Western blot result for infection with HIV? A. Band at p24 B. Band at gp60 C. Bands at p24 and p31 D. Bands at p24 and gp120

D To be considered positive by Western blot testing, bands must be found for at least two of the following three HIV proteins: gp41, p24, and gp120 or 160. The p24 band denotes antibody to a gag protein. The gp160 is the precursor protein from which gp120 and gp41 are made; these are env proteins.

What constitutes a diagnosis of viral hepatitis? A. Abnormal test results for liver enzymes B. Clinical signs and symptoms C. Positive results for hepatitis markers D. All of these options

D To diagnose a case of hepatitis, the physician must consider clinical signs as well as laboratory tests that measure liver enzymes and hepatitis markers.

The absorbance of a sample measured by ELISA is greater than the highest standard. What corrective action should be taken? A. Extrapolate an estimated value from the highest reading B. Repeat the test using a standard of higher concentration C. Repeat the assay using one half the volume of the sample D. Dilute the test sample

D Usually when a test sample reads at a value above the highest standard in an ELISA test, it is diluted and measured again. In those instances where no additional clinical value can be obtained by dilution, the result may be reported as greater than the highest standard (citing the upper reportable limit of the assay).

Which complement component is found in both the classic and alternative pathways? A. C1 B. C4 C. Factor D D. C3

D. C3 is found in both the classic and alternative (alternate) pathways of the complement system. In the classic pathway, C3b forms a complex on the cell with C4b2a that enzymatically cleaves C5. In the alternative pathway, C3b binds to an activator on the cell surface. It forms a complex with factor B called C3bBb which , like C4b2a3b, can split C5.

From the following, identify a specific component of the adaptive immune system that is formed in response to antigenic stimulation: A. Lysozyme B. Complement C. Commensal organisms D. Immunoglobulin

D. Immunoglobulin is a specific part of the adaptive immune system and is formed only in response to a specific antigenic stimulation. Complement, lysozyme, and commensal organisms all act nonspecifically as a part of the adaptive immune system. These three components do not require any type of specific antigenic stimulation.

Which markers are found on mature, peripheral helper T cells? A. CD 1, CD 2, CD 4 B. CD 2, CD 3, CD 8 C. CD 1, CD 3, CD 4 D. CD 2, CD 3, CD 4

D. Mature. peripheral helper T cells have the CD 2 (E rosette), CD 3 (mature T cell), and CE 4 (helper) markers.


Conjuntos de estudio relacionados

Algebra Module 3 linear equations

View Set

US History: Q3 - Lesson 4 - Assigment 8_Industrial Lifestyle: Trends

View Set

A Porra do Cursinho Inteiro (6 de Junho - 30 de Agosto)

View Set

principles of accounting chapter 7

View Set

APWH Chapter 24, APWH Chapter 23, AP Ways of the World: Chapter 22

View Set

Chapter 6 - Project Time Management

View Set

Florida Statutes, Rules, and Regulations Pertinent to Health Insurance

View Set